Sei sulla pagina 1di 97

Orthopaedics Past Year Questions

Batch 19 CA 1
1) A) Classification of OPEN FRACTURE
B) List the principle and procedure of “wound Debridement “in an open
fracture
C) List 4 complication of open fracture
-Gustilo - Anderson classification

Type I : wound small < 1cm long, little soft tissue damage,
minimal contamination, fracture not comminuted
Type II : wound 1-5 cm long, mod. soft tissue damage,
moderate contamination, fracture may be comminuted
Type III : extensive damage to skin, soft tissue & neuro-
vascular structures, wound contaminated, fracture comminuted
IIIA - loss of skin and muscles
IIIB - loss of periosteum
IIIC - associated neuro-vascular injury

B) List the principle and procedure of “wound Debridement “in an open fracture

Wound debridement
❖ Should be thorough

❖ Use plenty of saline for washing the wound clean

❖ Debride the wound till the wound is rendered absolutely clean and
devoid of all foreign particles

- Procedure
● Tourniquet not used : kept ready
● Preparation of part
● Sample for culture taken before starting the debridement & after
completion
● Irrigate wound with saline : minimum 5 liters
● Final irrigation with antibacterial solution
C) List 4 complication of open fracture
-General

• Crush syndrome (Traumatic Rhabdomyolysis or Bywaters' syndrome)

• DVT and PE

• Tetanus

Early Delayed Late

• Gas gangrene

- Infection - AVN
Bone - Delayed - Malunion
union - Nonunion

- Haemartrosis - Infection - Instability


Joints - Ligament - Sudeck’s - Stiffness
injury Osteodystrophy - Sudeck’s
Osteodystrophy
- Blisters - Bed sores - Myositis
Soft tissue - Injury to muscles - Tendonitis ossificans
Tendons - Tendon rupture
Nerves - Nerve
Vessels – Volkmann’s entrapment
ish.
- Volkmann’s
contracture

• Fat embolism

2) A) Briefly explain the stages of healing of fracture


B) Define non-union fracture
C) List 6 causes of non-union fracture
Healing of a fracture begins as soon as it breaks, provided the conditions are favorable
Stage 1
Haematoma formation
Due to tear of endosteal vessels
and periosteal arterioles
Haematoma around # site

• Bone in the immediate


• vicinity of the fracture

• dies - undergoes necrosis

Stage 2
Subperiosteal & endosteal
cellular proliferation
• Cells - deep layer of
periosteum & medullary
canal proliferate : form
osteoblasts : lays down
intercellular substance
• Haematoma gets pushed
aside by proliferating cells
& gets resorbed

Stage 3
• Formation of soft callus
Cellular tissue from both ends
differentiate into osteoblasts &
chondroblasts and form
intercellular martix
(collagen & polysaccharides)
with calcium salt
• Immature bone of fracture callus (woven bone)
Provides: stability to the fracture
Felt : as a hard mass around the fracture
Seen : radiologically, as the first sign of
fracture
➢ union
Stage 4
Formation of hard callus
Woven bone + osteoblastic activity
Mature bone
(has a lamellar structure)

Stage 5
Stage of remodeling
• At the fracture site - new bone
➢ forms a bulbous swelling around fracture
➢ medullary canal closed
• Redesigning of bone takes place along
lines of stress
➢ bone mass reduces
➢ medullary canal opens
This is called remodeling of the
bone : takes 1- 2 years

B) Define non-union fracture


Clinical, radiological & biological failure in healing of a
fracture despite twice the normal
duration required for “that fracture” to heal ( based on site of
fracture, type of bone, age
of the patient etc.)

C) List 6 causes of non-union fracture


• avascular necrosis (the blood supply was interrupted by the fracture)
• the two ends are not apposed (that is, they are not next to each other)
• infection (particularly osteomyelitis)
• the fracture is not fixed (that is, the two ends are still mobile)
• soft-tissue imposition (there is muscle or ligament covering the broken ends and
preventing them from touching each other

Batch 19 CA 2
1) A) Define “orthosis” and give an example of it
B) Classify bone grafts and state the indication for bone grafts
C)Briefly describe about Thomas splint
D) List the 4 sites of pin insertion in skeletal Traction
a) Define ‘orthosis’ and give an example of it.
Orthosis is an external applied device used to modify structure and function , to increase
stability of the spine or extremity following an or disease .

b)Classify bone grafts and state the indication of bone graft .

Classification
Source
i)Human bone
Autogenous
Allograft
ii)Bone substitute
Xenograft
Allograft
Content
Whole bone , cortical , cancellous

Vascularity
Vascularized ,non-vascularized

C )Briefly discuss about Thomas splint

Thomas splint is a metal splint for fractures of the arm or leg that consists of a
ring at one end to fit around the upper arm or leg and two metal shafts extending
down the sides of the limb in a long U with a crosspiece at the bottom where
traction is applied.It is mostly use for injurt of the hip and tight injuries .

d)List the 4 sites of pin insertion in skeletal traction .

Calcaneus , femoral ,tibial and olecranon .

2) A)Enumerate the pathophysiology of diabetic foot ulcer


B) Describe the ‘diabetic foot care’ in diabetic patient

i)Physiopathology-hypoxia , ischemia
ii)Neuropathy
-Sensory: Patient does not feel pain , unnoticed trauma , callous
formation , damage beneath callous , serious cavity and pus then ulcer
formation.
-Motor : Weakness of intrinsic muscle , wasting of intrinsic muscle ,
deform foot,abnormal gait , then ulcer .
iii)Autonomic : Decrease sweat , dry skin,crack , infection , then ulcer.
iv)Immune dysfunction
v)Vasculopathy-Microangiopathy results in decrease blood flow to distal
extreme causing delay in healing .

b) Describe the ‘diabetic foot care ‘ in diabetic patient .

DO Don’t
Check everyday for cuts,blister , redness Never use heating pat or electric blankets
or swelling
Wash & dry the foot Test water with hand before putting leg
Change socks everyday Do not wear tight socks
Clean the wound everyday Do not walk barefoot
Use protective foot wear Do not try to remove corns or calluses
Trim the nail straight Do not let your feet get wet .

3) A) Classify non-union fracture and classify it


B) Describe the management of non-union fracture

A) Classify non-union fracture and classify it

Nonunion being divided into hypertrophic and atrophic types. In hypertrophic non-
union ,the bone ends are enlarged, suggesting that osteogenesis is still active but not quite
capable of bridging the gap. In atrophic non-union, osteogenesis seems to have ceased.
The bone ends are tapered or rounded with no suggestion of new bone formation.

B) Describe the management of non-union fracture


Non-union fracture can be managed by conservative and operative.
Conservative :non-union is occasionally symptomless, needing no treatment or, at most, a
removable splint. Even if symptoms are present, operation is not the only answer; with
hypertrophic non-union, functional bracing may be sufficient to induce union, but
splintage often needs to be prolonged. Pulsed electromagnetic fields and low-frequency,
pulsed ultrasound can also be used to stimulate union.
Operative :With hypertrophic non-union and in the absence of deformity, very rigid
fixation alone (internal or external) may lead to union. With atrophic non-union, fixation
alone is not enough. Fibrous tissue in the fracture gap, as well as the hard, sclerotic bone
ends is excised and bone grafts are packed around the fracture. If there is significant ‘die-
back’, this will require more extensive excision and the gap is then dealt with by bone
advancement using the Ilizarov technique.

Batch 19 CA 3
Orthopedics
1) A) classify supracondylar fracture of humerus
B) State 4 complications of supracondylar fracture of humerus
c) Discuss the management of supracondylar fracture of humerus
2) A) Classify supracondylar fracture of humerus

Gartland classification system

Type Description[2]

I Non-displaced

II Angulated with intact posterior cortex

IIA Angulation
IIB Angulation with rotation

III Complete displacement but have perisosteal (medial/lateral) contact

IIIA Medial periosteal hinge intact. Distal fragment goes posteromedially

IIIB Lateral periosteal hinge intact. Distal fragment goes posterolaterally

IV Periostial disruption with instability in both flexion and extension

B) State 4 complications of supracondylar fracture of humerus


i) Neurovascular injury.
ii) Volkmann's contracture
iii) Malunion
iv) Compartment syndrome

c) Discuss the management of supracondylar fracture of humerus


Type I - Undisplaced or minimally displaced fractures can be treated by using an above
elbow spint in 90 degrees flexion for 3 weeks. Orthopaedic cast and extreme flexion
should be avoided to prevent compartment syndrome and vascular compromise. In case
the varus of the fracture site is more than 10 degrees when compared to the normal
elbow, closed reduction and percutaneous pinning using X-ray image
intensifier inside operating theater is recommended. In one study, for those children who
was done percutaneous pinning, immobilisation using a posterior splint and an arm
sling has earlier resumption of activity when compared to immobilisation using collar and
cuff sling. Both methods gives similar pain scores and activity level at two weeks of
treatment.
Type II - Gartland Type II fractures requires closed reduction and casting at 90 degrees
flexion. Percutaneous pinning is required if more than 90 degrees flexion is required to
maintain the reduction. Closed reduction with percutaneous pinning has low complication
rates. Closed reduction can be done by applying traction along the long axis of the
humerus with elbow in slight flexion. Full extension of the elbow is not recommended
because the neurovascular structures can hook around the proximal fragment of the
humerus. If the proximal humerus is suspected to have pierced the brachialis muscle,
gradual traction over the proximal humerus should be given instead. After that, reduction
can be done through hyperflexion of the elbow can be done with the olecranon pushing
anteriorly. If the distal fragment is internally rotated, reduction maneuver can be applied
with extra stress applied over medial elbow with pronation of the forearm at the same
time.
Type III and IV - Gartland III and IV are unstable and prone to neurovascular injury.
Therefore, closed or open reduction together with percutaneous pinning within 24 hours
is the preferred method of management with low complication rates. Straight arm lateral
traction can be a safe method to deal with Gartland Type III fractures. Although Gartland
Type III fractures with posteromedial displacement of distal fragment can be reduced
with closed reduction and casting, those with posterolateral displacement should
preferably be fixed by percutaneous pinning.

3) A) discuss stages of fracture healing


B) Define and classify non-union
C) Discuss management of non- union fracture
Fracture healing can be divided into two types:
 primary or direct healing by internal remodeling;
 secondary or indirect healing by callus formation.
The former occurs only with absolute stability and is a biological process of osteonal
bone remodeling. The latter occurs with relative stability (flexible fixation methods). It is
very similar to the process of embryological bone development and includes both
intramembraneous and endochondral bone formation. In diaphyseal fractures, it is
characterized by the formation of callus.
Bone healing can be divided into four stages:
 inflammation;
 soft callus formation;
 hard callus formation;
 remodeling.
Although the stages have distinct characteristics, there is a seamless transition from one
stage to another; they are determined arbitrarily and have been described with some
variation.

Inflammation
After fracture, the inflammatory process starts rapidly and lasts until fi brous tissue,
cartilage, or bone formation begins (1–7 days postfracture). Initially, there is hematoma
formation and inflammatory exudation from ruptured blood vessels. Bone necrosis is
seen at the ends of the fracture fragments. Injury to the soft tissues and degranulation of
platelets results in the release of powerful cytokines that produce a typical inflammatory
response, ie, vasodilatation and hyperemia, migration and proliferation of
polymorphonuclear neutrophils, macrophages, etc. Within the hematoma, there is a
network of fibrin and reticulin fibrils; collagen fibrils are also present. The fracture
hematoma is gradually replaced by granulation tissue. Osteoclasts in this environment
remove necrotic bone at the fragment ends.
The inflammation stage. Formation of hematoma resolving into granulation tissue with
the typical inflammatory cascade.

Soft callus formation


Eventually, pain and swelling decrease and soft callus is formed. This corresponds
roughly to the time when the fragments are no longer moving freely, approximately 2–3
weeks postfracture.

The soft callus stage. Intramembraneous ossification forming bone cuffs away from the
fracture gap. Replacement of the granulation tissue elsewhere in the callus by fibrous
tissue and cartilage, and ingrowth of vessels into the calcified callus. This starts at the
periphery and moves towards the center.

 At the end of soft callus formation, stability is adequate to prevent shortening,


although angulation at the fracture site may still occur.
The soft callus stage is characterized by the growth of callus. The progenitor cells in the
cambial layer of the periosteum and endosteum are stimulated to become osteoblasts.
Intramembraneous, appositional bone growth starts on these surfaces away from the
fracture gap, forming a cuff of woven bone periosteally, and filling the intramedullary
canal. Ingrowth of capillaries into the callus and increased vascularity follows. Closer to
the fracture gap, mesenchymal progenitor cells proliferate and migrate through the callus,
differentiating into fibroblasts or chondrocytes, each producing their characteristic
extracellular matrix and slowly replacing the hematoma

Hard callus formation


When the fracture ends are linked together by soft callus, the hard callus stage starts and
lasts until the fragments are firmly united by new bone (3–4 months). As
intramembraneous bone formation continues, the soft tissue within the gap undergoes
endochondral ossification and the callus is converted into rigid calcified tissue (woven
bone). Bone callus growth begins at the periphery of the fracture site, where the strain is
lowest. The production of this bone reduces the strain more centrally, which in turn forms
bony callus. Thus, hard callus formation starts peripherally and progressively moves
towards the center of the fracture and the fracture gap. The initial bony bridge is formed
externally or within the medullary canal, away from the original cortex. Then, by
endochondral ossification, the soft tissue in the gap is replaced by woven bone that
eventually joins the original cortex.

The hard callus stage. Complete conversion of callus into calcified tissue through
intramembraneous and endochondral ossification.
Remodeling
The remodeling stage begins once the fracture has solidly united with woven bone. The
woven bone is then slowly replaced by lamellar bone through surface erosion and
osteonal remodeling. This process may take anything from a few months to several years.
It lasts until the bone has completely returned to its original morphology, including
restoration of the medullary canal.

The remodeling stage. Conversion of woven bone into lamellar bone through surface
erosion and osteonal remodeling.
B) Define and classify non-union

 A non-union
- an arrest in the fracture repair process
o -progressive evidence of non healing of a fracture of a bone
o -a delayed union is generally defined as a failure to reach bony union by 6 months post-injury
 -this also includes fractures that are taking longer than expected to heal (ie. distal radial fractures)
o -large segmental defects should be considered functional non-unions
o

Classification

 Types of non-union
a. septic non-union
b. Pseudoarthrosis
c. hypertrophic nonunion
i. caused by inadequate immobilization with adequate blood supply
ii. type 2 collagen is elevated
iii. typically heal once mechanical stability is improved
d. atrophic nonunion
i. caused by inadequate immobilization and inadequate blood supply
e. oligotrophic nonunion
i. produced by inadequate reduction with fracture fragment displacement

C) Discuss management of non-union fracture

Nonsurgical Treatment

Some nonunions can be treated nonsurgically. The most common nonsurgical treatment
is a bone stimulator. This small device delivers ultrasonic or pulsed electromagnetic
waves that stimulate healing The patient places the stimulator on the skin over the
nonunion from 20 minutes to several hours daily. This treatment must be used every day
to be effective.

An external bone stimulator is applied to the skin overlying the nonunion.

Surgical Treatment

Surgery is needed when nonsurgical methods fail. You may also need a second surgery if
the first surgery failed. Surgical options include bone graft or bone graft substitute,
internal fixation, and/or external fixation.
 Bone Graft. During this procedure, bone from another part of the body at
the fracture site to "jump start" the healing process. A bone graft provides a
scaffold on which new bone may grow. Bone grafts also provide fresh bone
cells and the naturally occurring chemicals the body needs for bone healing.

During the procedure, a surgeon makes an incision and removes (harvests)


pieces of bone from different areas on the patient. These are then
transplanted to the nonunion site. The rim of the pelvis or "iliac crest" is
most often used for harvesting bone. Although harvesting the bone may be
painful, the amount of bone removed usually does not cause functional,
structural, or cosmetic problems.

 Bone graft is taken from the back of the pelvis and placed at the nonunion site.

 Allograft (cadaver bone graft). An allograft (cadaver) bone graft avoids


harvesting bone from the patient, and therefore, decreases the pain involved with
treating the nonunion. Like a traditional bone graft, it provides scaffolding for the
patient's bone to heal across the area of the nonunion. As time goes on, the
patient's bone replaces the cadaver bone. Although there is a theoretical risk of
infection, the cadaver bone graft is processed and sterilized to minimize this risk.
 Bone graft substitutes and/or osteobiologics. As with allografts, bone graft
substitutes avoid the bone harvesting procedure and related pain. Although bone
graft substitutes do not provide the fresh bone cells needed for normal healing,
they do provide a scaffold chemicals needed for growth.
 Depending on the type of nonunion, any of the above materials, or a combination
of materials, may be used to fix the nonunion.
 Bone grafts (or bone graft substitutes) alone provide no stability to the fracture
site. Unless the nonunion is inherently stable, you may also need more surgical
procedures (internal or external fixation) to improve stabililty.
 Internal Fixation. Internal fixation stabilizes a nonunion. The surgeon attaches
metal plates and screws to the outside of the bone or places a nail (rod) in the
inside canal of the bone.

If a nonunion occurs after internal fixation surgery, another internal fixation


surgery may be needed to increase stability. The surgeon may use a more rigid
device, such as a larger rod (nail) or a longer plate. Removing a previously
inserted nail and inserting a larger one (exchange nailing) increases stability and
stimulates healing within the bone. Internal fixation can be combined with bone
grafting to help stability and stimulate healing.
 External fixation. External fixation stabilizes the injured bone, as well. The
surgeon attaches a rigid frame to the outside of the injured arm or leg. The frame
is attached to the bone with wires or pins. External fixation may be used to
increase the stability of the fracture site if instability helped cause the nonunion.
External fixation can treat nonunions in a patient who also has bone loss and/or
infection.

An external fixator made of pins and wires

4) A) Classify Diabetic foot ulcer


B) Discuss on diabetic foot care
C) Describe “foot at risk” for DFU
A) Classify diabetic foot ulcer
B) Discuss on diabetic foot care
C) Describe “foot at risk” for DFU

-Absence of protective sensation due to peripheral neuropathy


-Arterial insufficiency
-Foot deformity and callus formation resulting in focal areas of high pressure
-Autonomic neuropathy causing decreased sweating and dry, fissured skin
-Limited joint mobility
-Obesity
-Impaired vision
-Poor glucose control leading to impaired wound healing
-Poor footwear that causes skin breakdown or inadequately protects the skin from high
pressure and shear forces
-History of foot ulcer or lower extremity amputation
Batch 15 PE4
Short Answer Questions:
1. Osteoarthritis
(a) Clinical features
(b) Causes of secondary Osteoarthritis,
(c) Radiological diagram of Osteoarthritis
(d) Principles of management

(a) Clinical features


- Pain
- swollen joints
- joint stiffness
- joint creaking
- loss of range of motion.

(b) Causes of secondary Osteoarthritis,


- obesity
- repeated trauma or surgery to the joint structures
- abnormal joints at birth (congenital abnormalities)
- gout
- diabetes
- other hormone disorders
(c) Radiological diagram of Osteoarthritis

(d) Principles of management


2. Acute Osteomyelitis
(a) Clinical features
(b) Radiological features
(c) Complications
(d) Principles of management

(a) Clinical features

 Fever or chills

 Irritability or lethargy in young children

 Pain in the area of the infection

 Swelling, warmth and redness over the area of the infection

(b) Radiological features

The earliest changes are seen in adjacent soft tissues +/- muscle outlines with swelling
and loss or blurring of normal fat planes. An effusion may be seen in an adjacent joint.
Others features are:
 regional osteopaenia

 periosteal reaction/thickening (periostitis): variable, and may appear aggressive


including formation of a Codman's triangle 6

 focal bony lysis or cortical loss

 endosteal scalloping 8

 loss of bony trabecular architecture


 new bone apposition

 eventual peripheral sclerosis

In chronic or untreated cases eventual formation of


a sequestrum, involucrum or cloaca may be seen.

(c) Complications

 Bone death (osteonecrosis). An infection in your bone can impede blood


circulation within the bone, leading to bone death. Your bone can heal after surgery
to remove small sections of dead bone. If a large section of your bone has died,
however, you may need to have that limb surgically removed (amputated) to
prevent spread of the infection.

 Septic arthritis. In some cases, infection within bones can spread into a nearby
joint.

 Impaired growth. In children, the most common location for osteomyelitis is in the
softer areas, called growth plates, at either end of the long bones of the arms and
legs. Normal growth may be interrupted in infected bones.

 Skin cancer. If your osteomyelitis has resulted in an open sore that is draining pus,
the surrounding skin is at higher risk of developing squamous cell cancer.

(d) Principles of management


Batch 17 Year 4 Continuous Assessment
Multiple Choice Questions:
1. Torticolis
2. Osteochondroma
3. Scaphoid fracture
4. Acute anterior shoulder dislocation
5. Synovial fluid analysis
6. Wagner Classification 3
7. Wrist fracture
EMI
1. Spondylosis
2. Median nerve
3. Bone graft
Short Answer Questions:
1. (a) Diagnosis of Anterior Shoulder Dislocation
Presenting complaints:
 Shoulder abducted and the elbow supported with opposite hand.
 History of a fall on an out-stretched hand followed by pain and inability to
move the shoulder.
On examination:
 Arm abducted.
 The normal round contour of the shoulder joint is lost, and it becomes
flattened.
 Fullness below the clavicle due to the displaced head.
The diagnosis is easily confirmed on an anteroposterior X-ray of the shoulder. An
axillary view is sometimes required.

(b) Describe 4 clinical tests to elicit the diagnosis


1. Dugas' test: Inability to touch the opposite shoulder.
2. Hamilton ruler test: Because of the flattening of the shoulder, it is possible to
place a ruler on the lateral side of the arm. This touches the acromion and lateral
condyle of the humerus simultaneously.
3. Load and shift test
This test can be performed with the patient in sitting or in supine. In sitting,
the patient’s arm rests on the thigh with the examiner to their side and slightly
behind. One hand is used to stabilise the scapula and the other is placed on the
shoulder. The thumb is positioned over the posterior humeral head and fingers
over the anterior humeral head. The humerus is loaded by pushing the
humeral head into a neutral position within the fossa. Whilst maintaining the
humeral head in this position, humerus is shifted forwards by applying an
anterior force, to asses anterior instability. Some movement is normal, but
should not be more than 25% of the humeral head.

 Grade 1 instability is present if there is a shift between 25% and 50%,


 Grade 2 is present with more than a 50% shift with spontaneous
reduction when the force is stopped,
 Grade 3 is present when the humeral head shows more than a 50%
shift without spontaneous reduction and remains dislocated. A
combination of laxity and a reproduction of the symptoms determines
a positive or negative result.

4. Apprehension, Relocation and anterior release tests:


The patient is in a supine position, with the shoulder in 90° of abduction and
maximal lateral rotation. A positive apprehension test occurs if the patient
either looks apprehensive or resists further movement. To differentiate
apprehension from other potential conditions, the relocation test is used. Start
position is the same as that for the apprehension test, then an anterior -
posterior force is applied to the shoulder to relocate the humerus in the fo ssa.
The apprehension will decrease in the case of shoulder instability. The final
test is the release test, where the posteriorly directed force applied in the
relocation test is removed. The result is considered positive if the patient’s
apprehension returns.

5. Anterior drawer test:


The patient is in a supine position and the affected shoulder over the edge of
the table. The patient’s arm should be relaxed. Position the arm in a combined
midrange abducted position with forward flexion and lateral rotation. The
stabilising hand is placed on the scapula so that the fingers and thumb secure
the scapula at the spine of the scapula and the coracoid. The patient’s arm is
pulled anteriorly to apply a gliding force to the glenohumeral joint. If an
audible click is heard during the movement, the glenoid labrum may be torn,
or the joint may be sufficiently lax to allow the humeral head to glide over the
glenoid labrum rim.

(c) List 2 immediate and 2 late complications of the mentioned diagnosis.


Early :
 Nerve vascular injury
 Axillary nerve injury
 Associated fracture of neck of humerus, greater and lesser tuberosities
Late :
 Recurrent dislocations
 Avascular necrosis of head of humerus
 Heterotropic calcification

2. (a) Define and draw a Colles fracture


This is a fracture at the distal end of the radius, at its cortico-cancellous junction
(about 2 cm from the distal articular surface), in adults, with typical displacement. It
is the commonest fracture in people above forty years of age, and is particularly
common in women because of postmenopausal osteoporosis. It nearly always results
from a fall on an out-stretched hand.

(b) Management of Colles fracture


 Treatment of Colles' fracture is essentially conservative.
 For an undisplaced fracture, immobilisation in a below-elbow plaster cast for
six weeks is sufficient.
 For displaced fractures, the standard method of treatment is manipulative
reduction followed by immobilisation in Colles' cast.
 For Impacted or Comminuted Colles’ Fracture, manipulation and plaster
immobilization alone may be insufficient. The fracture can sometimes be
reduced and held with percutaneous wires, but if impaction is severe even this
may not be enough to maintain length; in that case, an external fixator is used
(c) List 4 late complications of Cole’s fracture.
i. Malunion
ii. Delayed union and non-union
iii. Stiffness
iv. Tendon rupture

3. (a) Describe the management of clavicular fracture.


(b) Draw and define Barton’s fracture
(c) List 2 complications of scaphoid fracture.
(d) Discuss the management of Galleazzi’s fracture.

Most clavicular fracture heal without surgical operation. A simple arm sling is
usually used for comfort immediately after the break and to keep your arm and shoulder
in position while the injury heals

Surgical treatment ,Open reduction and internal fixation is indicated when


there is:
1) Non-union
2) Neurovascular involvement
3) Comminuted fracture
4) Open fracture
5) Floating shoulder

(3b)Draw and define Barton ‘s fracture .

A Barton's fracture is an intra-articular fracture of the distal radius with


dislocation of the radiocarpal joint

(3c)List 2 complication of scaphoid fracture .


 Avascular necrosis
 Non Union

(3b)Discuss the management of Galeazzi’s fracture .


Relocation of the distal radial-ulna joint together with precise reduction of the radial
fracture which is rigidly fixed . This can be done through anterior approach to expose
fracture then put plate on volar aspect of distal radial shalf .

4. Discuss the clinical features, radiological classification (with diagrams),


4 complications and treatment options of Supracondylar fractures.
Clinical Feature
 absence of radial pulse.
 ischaemia of hand: pale, cool.
 severe swelling in forearm and or elbow.
 skin puckering or anterior bruising.
 open injury.
 neurological injury.

Radiological classification

4 complications
Malunion –cubitus cavus
Volksman’s ischemia
Pin tract infection
Loss of elbow motion

Treatment Option
Type 1 –Plaster of Paris back slap with elbow in flexion for 3 weeks.
Type 2a-Closed reduction +/- percutaneous pinning with crossed K-wire
Type 2b and 3 - Closed reduction +percutaneous pinning with crossed K-wire
Open reduction indication- failure of closed reduction ,open supracondylar
fracture ,comminuted fracture .
5. Vertebrae injury
(a) Classify vertebral injuries
AO classification (of thoracolumbar spinal fracture)
Type A: Compression injuries
Type B: Distraction injuries
Type C: Torsional injury
(b) Define spinal shock
combination of areflexia/hyporeflexia and autonomic dysfunction that
accompanies spinal cord injury
(c) Differentiate between Upper Motor Neurone Lesion and Lower Motor Neurone
Lesion of bladder.
*change defecation to urinary. EAS to external urethral sphincter.
Upper vs Lower Motor Neuron lesion of the bladder
Suprasacral Infrasacral
(reflex or spastic type) (areflexic or flaccid
type)
Level of injury Spinal cord above Conus or cauda equina
conus
Urination reflex Intact Disrupted
Voluntary control over Absent Absent
urination
External Urethral Hypertonic with Poor or absent
Sphinter tone anorectal dyssynergia
Bulbocavernosus reflex Positive Negative

(d) Importance of bulbocavernous reflex.


The test involves monitoring internal/external anal sphincter contraction in
response to squeezing the glans penis or clitoris, or tugging on an indwelling
Foley catheter.
The reflex is spinal mediated and involves S2-S4. The absence of the reflex in a
person with acute paralysis from trauma indicates spinal shock whereas the
presence of the reflex would indicate spinal cord severance. Typically this is one
of the first reflexes to return after spinal shock. Lack of motor and sensory
function after the reflex has returned indicates complete SCI. Absence of this
reflex in instances where spinal shock is not suspected could indicate a lesion or
injury of the conus medullaris or sacral nerve roots.
6. Free fluid in knee
(a) List the causes
(b) Discuss the clinical tests
(c) Synovial fluid analysis.
(a) List the causes

o overuse of the joint (repetitive injury)


o torn ligament or cartilage (meniscus)
o broken bone
o bacterial infection
o prepatellar bursitis
o osteoarthritis or rheumatoid arthritis
o gout or pseudogout
o cysts or tumors
(a) Discuss the clinical tests
Patellar Tap Test
 The patient is lying in supine with the leg extended. The physiotherapist
puts pressure on the proximal side of the knee in an effort to squeeze the
fluid out of the suprapatellar pouch. The fluid can be moved under the
patella while maintaining the pressure on the suprapatellar pouch; the
therapist uses his/her other hand to press up on the medial and lateral
recesses forcing the fluid under the patella . Tapping down the patella
with the index to create an upward and downward movement and a
palpable ‘click’ as the patella hits the underlying femur.
 If the test is negative the femur and the patella are already in contact.
A positive test is when the patella can be felt to move down through the
fluid and rebounds on the patella. The test can be false positive, therefore
we must always test both sides to compare.
Fluid Displacement Test

 The patient in supine, with the knee in an extended position. The


physiotherapist strokes upwards with the edge of the hand on the medial
side of the knee to milk the fluid 10 cm proximal of the patella into the
lateral compartment, and continues pushing the fluid downwards on the
lateral side.
 The test is positive if the physiotherapist sees fluid moving towards the
medial side of the knee.
Fluid Wave Test

 Is used when the effusion is less than 30-50 cc. The patient is in supine,
the physiotherapist presses his/her fingers in both parapatellar gutters.
Because there’s a pressure from below upward, the gutters are emptied.
The patient is asked to stand while the physiotherapist keeps his/her
fingers in the parapatellar gutters. If the physiotherapist releases his/her
fingers and the fluid comes back in the parapatellar gutters, it is a
positive sign.
(b) Synovial fluid analysis
Synovial fluid analysis is also known as joint fluid analysis. It helps diagnose
the cause of joint inflammation. It is performed when pain, inflammation, or
swelling occurs in a joint, or when there’s an accumulation of fluid with an
unknown cause.
The process of removing fluid from a joint is called arthrocentesis.

Synovial fluid sample will be sent to the laboratory for examination. The
colour and thickness of the fluid will be observed and red and white blood
cells will be assessed under a microscope. We will also look for crystals or
signs of bacteria and all the below item will be measured:

 glucose
 proteins
 uric acid
 lactic dehydrogenase (an enzyme that increases in cases of inflammation
and tissue damage)

The fluid sample will also be cultured to test for bacteria.

Expected Synovial Fluids per category


7. (a) Define Osteoporosis [1m]
(b) Mention 4 risk factors in women leading to osteoporosis. [1m]
(c) Mention 4 important investigations in diagnosing Primary Osteoporosis.
[2m]
(d) Mention 4 important groups of drugs with one example each that is used in
its management. [1m]
Osteoporosis is a condition characterized by a decrease in the density of bone,
decreasing its strength and resulting in fragile bones.
(b) Mention 4 risk factors in women leading to osteoporosis.
 Low oestrogen level
 Amenorrhea
 Diet low in calcium
 Cigarette smoking and alcohol consumption

(c) Mention 4 important investigations in diagnosing Primary Osteoporosis.


 Plain x-ray
 DEXA (T-score)
 Calcium level in body
 Peripheral smear

(d) Mention 4 important group of drugs with one example each that is used in its
management.
 Biphosphonates : Alendronate (Fosamax)
 Hormones : Eostrogen
 Denosumab (Prolia)
 Teriparatide (Forteo)

8. (a) Classify open fractures. [1.5m]


(b) What are the steps in ‘debridement’ of open fractures? [2.5m]
(c) Mention 4 general complications of open fractures. [1m]

Gustilo Grade Definition

Open fracture, clean wound, wound <1 cm in length, mild contamination, and mild soft
I
tissue involvement, bone not comminuted.

Open fracture, wound > 1 cm but < 10 cm in length , moderate contamination and
II
moderate soft tissue involvement, bone may comminuted.
Open fracture with adequate soft tissue coverage of a fractured bone despite extensive
IIIA soft tissue laceration or flaps, or high-energy trauma (gunshot and farm injuries)
regardless of the size of the wound

Open fracture with extensive soft-tissue loss and periosteal stripping and bone damage.
IIIB Usually associated with massive contamination. Will often need further soft-tissue
coverage procedure (i.e. free or rotational flap)

Open fracture associated with an arterial injury requiring repair, irrespective of degree
IIIC
of soft-tissue injury.

(b) What are the steps in “debridement” of open fractures.

1. Wound margin is excised


2. Wound is exposed.
3. Wound is extended.
4. Complete irrigation of wound with normal saline. (10-12 L)
(c) Mention 4 general complications of open fracture.

 Shock
 Crush Syndrome
 Pulmonary embolism
 Fat embolism

9. (a) What is the pathophysiology in the formation of diabetic foot ulcer? [2m]
(b) Mention 4 complications of diabetic foot ulcer. [1m]
(c) What are the principles in the management of a diabetic foot ulcer? [2m]
(b) 4 Complications of Diabetic Foot Ulcer

(c) Principles in Management of a Diabetic Foot Ulcer

The cornerstone of management of the diabetic foot is regular inspection and examination
of the foot. Risk categorisation as set out by the International Working Group on the
Diabetic Foot guides the frequency of visits and correlates well with ulcer incidence
(Table I). Treatment of the high-risk foot may be either non-surgical or surgical, but in
both instances requires a team approach from a variety of specialists working together to
salvage the foot at risk. At the same time, education of the patient and family members is
crucial and should be simple, relevant and consistent. The steps of successful
management include the following:
• prompt detection and intervention of the high-risk foot
• medical management of diabetes and comorbid conditions
• antibiotic coverage
• vascular work-up
• consultations with
• diabetologist
• infectious disease specialist
• foot and ankle surgeon
• vascular surgeon
• podiatrist
• prosthetist/orthotist
• physiotherapist
• wound care sister
• post-surgical surveillance/wound care
• lifelong multidisciplinary clinic attendance.

Increased plantar foot pressure with callus formation remains a risk factor for ulceration.
The removal of callus by a podiatrist, or a trained professional who is knowledgeable
about the pathology of the diabetic foot, can reduce plantar pressure by up to 30%. Both
neuropathic and ischaemic ulcers are frequently complicated by infection.

All non-viable tissue must be extensively debrided and any abscesses from the deep
compartments of the foot must be drained. It is extremely important to send tissue
specimens for culture, and in the case of suspected osteomyelitis, a bone biopsy is
essential for diagnosis. Immediate revascularisation must follow after debridement of
foot ulcers in the ischaemic foot if wound healing is to take place.

Surgical procedures involve either foot salvage surgery or amputation with rehabilitation
whenever possible. Surgery can be divided into elective surgical procedures, prophylactic
surgical procedures or emergency surgical procedures. All these procedures can also be
done in a minimally invasive way by means of arteriography with percutaneous
transluminal angioplasty and/ or stenting.

10. (a) What are the clinical features of acute osteomyelitis of the distal femur?
[1.5m]
(b) What are the radiological features in acute osteomyelitis? [1.5m]
(c) How is this condition managed? [2m]
- Pain (several weeks/months)
- Limping
- Swelling & Local Tenderness
- Muscle wasting
- Joint stiffness
- Body temperature slightly elevated (mild fever)
- Irritability
- Fatigue

(b) Radiological Features in Acute Osteomyelitis

In some instances, radiographic features are specific to a region or a particular type of


infection, for example:

 subperiosteal abscess
- is one of the more frequent complications of acute otomastoiditis and results
in coalescent mastoiditis extending through the external cortex of the mastoid sinus. This
can occur in any direction:
postauricular: common as the bone is particularly thin ("Macewen's triangle")
inferomedial: medial to the attachment of sternocleidomastoid can result in a Bezold's
abscess

 Brodie's abscess
- is an intraosseous abscess related to a focus of subacute
pyogenic osteomyelitis. Unfortunately, there is no reliable way radiographically to
exclude a focus of osteomyelitis. It has a protean radiographic appearance and can occur
at any location and in a patient of any age. It might or might not be expansile, have a
sclerotic or nonsclerotic border, or have associated periostitis.
 Pott's puffy tumour

- refers to a non-neoplastic complication of acute sinusitis. It is characterised by a


primarily subgaleal collection, subperiosteal abscess, and osteomyelitis. It is usually
related to the frontal sinus but is sometimes secondary to mastoid pathology.

Rarer aetiologies include trauma, intranasal cocaine and methamphetamine abuse,


and craniotomy.

 Sclerosing osteomyelitis of Garré


- is a specific type of chronic osteomyelitis. It mainly affects children and young adults. It
typically affects the mandible and is commonly associated with an odontogenic infection
resulting from dental caries.

Below are general features of osteomyelitis.

The earliest changes are seen in adjacent soft tissues +/- muscle outlines with swelling
and loss or blurring of normal fat planes. An effusion may be seen in an adjacent joint.

In general, osteomyelitis must extend at least 1 cm and compromise 30 to 50% of bone


mineral content to produce noticeable changes in plain radiographs. Early findings may
be subtle, and changes may not be obvious until 5 to 7 days in children and 10 to 14 days
in adults. After this time a number of changes may be noted:

 regional osteopaenia
 periosteal reaction/thickening (periostitis): variable, and may appear aggressive
including formation of a Codman's triangle 6
 focal bony lysis or cortical loss
 endosteal scalloping 8
 loss of bony trabecular architecture
 new bone apposition
 eventual peripheral sclerosis

In chronic or untreated cases eventual formation of


a sequestrum, involucrum or cloaca may be seen.

(c) Management of Osteomyelitis


11. (a) Classify open fractures. [1.5m]
(b) What are the steps in would debridement in open fractures [2m]
(c) What are the clinical features in fat embolism. [1.5m]
Type I: clean wound smaller than 1 cm in diameter, appears clean, simple fracture
pattern, no skin crushing.

Type II: a laceration larger than 1 cm but without significant soft-tissue crushing,
including no flaps, degloving or contusion. Fracture pattern may be more complex.

Type III: an open segmental fracture or a single fracture with extensive soft-tissue
injury. Also included are injuries older than eight hours. Type III injuries are
subdivided into three types:
Type IIIA: adequate soft-tissue coverage of the fracture despite high-energy trauma or
extensive laceration or skin flaps. Gunshot injury and barnyard injury also classified as
111 A regardless of size.

Type IIIB: inadequate soft-tissue coverage with periosteal stripping. Soft-tissue


reconstruction is necessary.

Type IIIC: any open fracture that is associated with vascular injury that requires repair.
B) steps in open fracture wound debridement

Skin
-Wound is extended
-Skin margins : trimmed till bleeding edges seen
-Skin retained as much as possible
-Once debrided, the extended portion may be closed
-Exposed bone is covered with muscle
-Skin closure decided based on contamination : preferably closed as delayed primary
procedure

Fascia
-Fasciae divided extensively so that circulation is not impeded.Fascia need not be closed
Muscle
-All dead muscle should be removed : nidus for infection. Recognised based on
color,bleeding and contractility.
Blood vessels
-Large bleeding vessels : tied meticulously
-Small vessels : clamped or cauterised : cauterisation : minimal
Nerves & tendons
-Cut nerves : left undisturbed
-Sheath of cut ends tagged : non- absorbable suture material : for identification later
-Repair of nerve usually done at a later date
-Primary repair done only when wound is clean & further dissection is not necessary for
repair
Bone debridement
-Fracture ends : cleaned & reduced
-Small fragments totally devoid of
blood supply : removed
Stabilization of bone
-Reduces infection rate especially in type II & III fractures
-Type I may be stabilised by POP casts
-Type II & III stabilised by external fixation

C) clinical features of fat embolism


Cardiopulmonary
-Early persistent tachycardia ,Tachypnea, dyspnea and hypoxia due toventilation-
perfusion abnormalities (after 12-72 hrs)
-Febrile with high-spiking temperatures
-Skin :Reddish-brown nonpalpable petechiae - diagnostic over upper body particularly in
axillae within 24-36 hrs.
-Neurology :Central nervous system dysfunction ,agitated delirium progressing to
stupor,seizures and coma . Frequently unresponsive to correction of hypoxia
-Eye: Retinal hemorrhages intra-arterial fat globules visible - funduscopic examination
Subconjunctival hemorrhages and petechiae
-Oral :Haemorhages and petechiae

12. (a) What are the clinical features in developmental dysplasia of the hip? [1.5m]
(b) Draw a diagram explaining its radiological features. [1.5m]
(c) How would you manage this condition in a newborn? [2m]
A) clinical features in development dysplasia of hip
-The leg may appear shorter on the side of the dislocated hip

-The leg on the side of the dislocated hip may turn outward

-The folds in the skin of the thigh or buttocks may appear uneven

-The space between the legs may look wider than normal

B)management of ddh in newborn

-A dynamic flexion-abduction orthosis

-splint treatment

-surgery is indicated for those who do not respond to early splint

-closed reduction with adductor or psoas tenotomy


-Pavlik harness

13. (a) What are the clinical features of Osteosarcoma of the distal femur? [2m]
(b) Draw a diagram to show its radiological features. [2m]
(c) Outline the options in its management. [1m]
(a) What are the clinical features of Osteosarcoma of the distal femur? [2m]

 Pain

 Swelling

 Occasional pyrexia

 Pulmonary mets

(b) Draw a diagram to show its radiological features. [2m]

 soft-tissue mass
 tumour matrix ossification/calcification

(c) Outline the options in its management.

 Surgery
 Radiation therapy
 Chemotherapy
 Rotationplasty
14. (a) What are the clinical features in Osteoarthritis of the knee joint? [2m]
(b) Draw a diagram to show its radiological features. [1m]
(c) Outline its management. [2m]

Pain. Your joint may hurt during or after movement.

Tenderness. Your joint may feel tender when you apply light pressure to it.

Stiffness. Joint stiffness may be most noticeable when you wake up in the morning or
after a period of inactivity.

Loss of flexibility. You may not be able to move your joint through its full range of
motion.

Grating sensation. You may hear or feel a grating sensation when you use the joint.

Bone spurs. These extra bits of bone, which feel like hard lumps, may form around the
affected joint

(b) Draw a diagram to show its radiological features. [1m]

(c) Outline its management. [2m]


Osteoarthritis is a chronic (long-term) disease. There is no cure, but treatments are
available to manage symptoms. Long-term management of the disease will include
several factors:
 Managing symptoms, such as pain, stiffness and swelling

 Improving joint mobility and flexibility

 Maintaining a healthy weight

 Getting enough of exercise

Physical Activity
One of the most beneficial ways to manage OA is to get moving. While it may be hard to
think of exercise when the joints hurt, moving is considered an important part of the
treatment plan. Studies show that simple activities like walking around the neighborhood
or taking a fun, easy exercise class can reduce pain and help maintain (or attain) a healthy
weight.

Strengthening exercises build muscles around OA-affected joints, easing the burden on
those joints and reducing pain. Range-of-motion exercise helps maintain and improve
joint flexibility and reduce stiffness. Aerobic exercise helps to improve stamina and
energy levels and also help to reduce excess weight. Talk to a doctor before starting an
exercise program.

Weight Management
Excess weight adds additional stress to weight-bearing joints, such as the hips, knees, feet
and back. Losing weight can help people with OA reduce pain and limit further joint
damage. The basic rule for losing weight is to eat fewer calories and increase physical
activity.

Stretching
Slow, gentle stretching of joints may improve flexibility, lessen stiffness and reduce pain.
Exercises such as yoga and tai chi are great ways to manage stiffness.

Pain and Anti-inflammatory Medications


Medicines for osteoarthritis are available as pills, syrups, creams or lotions, or they
are injected into a joint. They include:

 Analgesics. These are pain relievers and include acetaminophen, opioids (narcotics)
and an atypical opioid called tramadol. They are available over-the-counter or by
prescription.

 Nonsteroidal anti-inflammatory drugs (NSAIDs). These are the most commonly


used drugs to ease inflammation and related pain. NSAIDs include aspirin, ibuprofen,
naproxen and celecoxib. They are available over-the-counter or by prescription.
 Corticosteroids. Corticosteroids are powerful anti-inflammatory medicines. They are
taken by mouth or injected directly into a joint at a doctor’s office.
 Hyaluronic acid. Hyaluronic acid occurs naturally in joint fluid, acting as a shock
absorber and lubricant. However, the acid appears to break down in people with
osteoarthritis. The injections are done in a doctor’s office.

Physical and Occupational Therapy


Physical and occupational therapists can provide a range of treatment options for pain
management including:
 Ways to properly use joints

 Heat and cold therapies

 Range of motion and flexibility exercises

 Assistive devices

Assistive Devices
Assistive devices can help with function and mobility. These include items, such as like
scooters, canes, walkers, splints, shoe orthotics or helpful tools, such as jar openers, long-
handled shoe horns or steering wheel grips. Many devices can be found at pharmacies
and medical supply stores. But some items, such as custom knee braces and shoe wedges
are prescribed by a doctor and are typically fitted by a physical or occupational therapist.

Natural and Alternative Therapies


Many people with OA use natural or alternative therapies to address symptoms and
improve their overall well-being. These include nutritional supplements, acupuncture or
acupressure, massage, relaxation techniques and hydrotherapy, among others.

Surgery
Joint surgery can repair or replace severely damaged joints, especially hips or knees. A
doctor will refer an eligible patient to an orthopaedic surgeon to perform the procedure.

Positive Attitude
Many studies have demonstrated that a positive outlook can boost the immune system
and increase a person's ability to handle pain.

15. (a) Define and classify closed fractures. [2m]


(b) What are the clinical features in a patient with fat embolism? [2m]
(c) Mention 2 investigations and their expected results that would help diagnose
fat embolism. [1m]
A closed fracture is also called a simple fracture. In a closed fracture, the broken
bone doesn't break your skin.
(b) What are the clinical features in a patient with fat embolism? [2m]

(c) Mention 2 investigations and their expected results that would help diagnose fat
embolism. [1m]
An otherwise unexplained increase in pulmonary shunt fraction alveolar-to-arterial
oxygen tension difference, especially if it occurs within 24-48 hours of a sentinel event
associated with fat embolism syndrome (FES), is strongly suggestive of the syndrome.
Thrombocytopenia, anemia, and hypofibrinogenemia are indicative of FES; however,
they are nonspecific.
Urinary fat stains are not considered to be sensitive or specific enough for diagnosing
FES or for determining the risk of it. Fat globules in the urine are common after
trauma. [10]
Preliminary investigations of the cytology of pulmonary capillary blood obtained from
a wedged pulmonary artery catheter revealed fat globules in patients with FES and
showed that this method may be beneficial in early detection of patients at risk.
In the future, genotyping for polymorphisms associated with increased susceptibility
to inflammatory stimuli may help identify those at risk for FES. Specific antibody
therapy targeting inflammatory molecules has not been useful.
16. (a) List 4 important differential diagnosis in a young adult male complaining of
acute post traumatic knee pain. [2m]
(b) Mention 2 important investigations with expected results required in its
diagnosis. [1m]
(c) Mention principles in management of any one of the differential diagnosis.
[2m]
a)
 Patellar dislocation
 Medial meniscal tear
 Lateral meniscal tear
 Posterior cruciate ligament injury
b) Full Blood Count and Radiography

17. (a) What are the clinical and radiological features of Colle’s fracture? [3m]
(b) Mention 4 late complications following Colle’s fracture. [3m]
a) Clinical features of Colle’s Fracture is a Dinner Fork Deformity and the radiological
features of it would be a fracture appears extra-articular and usually proximal to the
radioulnar joint. Dorsal angulation of the distal fracture fragment is present to a variable
degree.

b)
 Volkmann’s ischemia
 Radiocarpal arthrosis
 Finger stiffness
 Malposition-malunion
18. (a) What are the clinical features of various stages of Tuberculosis of the hip?
[2m]
(b) What are the radiological features of tuberculosis affecting the hip joint?
[2m]
(c) What are the principles in its management? [2m]

Management

Early diagnosis and effective chemotherapy are vital to save the joint. In the case of the
abduction deformity, traction on the other limb is also applied to stabilize the pelvis.
Traction relieves the muscle spasm, prevents or corrects deformity and subluxation,
maintains the joint space, minimizes the chances of development of migration of
acetabulum and permits close observation of the hip region. It also keeps the joint surfaces
apart; hence with an early start of mobilization exercises of the hip, functional range of
movements can be achieved.

Synovitis stage
To establish the diagnosis the patient should be subjected to USG examination; synovial
effusion can be aspirated and subjected for cytology, AFB smear and PCR examination. If
necessary, biopsy can be taken from diseased tissue to establish the diagnosis. Surgical
interventions usually are not required.

Early arthritis
MRI may show synovial effusion, osseous edema and areas of bone destruction. In addition
to traction and chemotherapy, analgesics supplementation is necessary till spasm of the
muscles is relieved. Nonweight bearing range of motion exercises are started whenever
patient is able to co-operate. Synovectomy and joint debridement are done with an aim to
reduce the disease tissue load and ascertain diagnosis. The prognosis in general is good.
Deformities are correctable, shortening is minimal and range of movements can be more
than functional depending upon how seriously exercises regimen is followed.

Advanced arthritis
In addition to the treatment as advocated above, arthrolysis of joint with joint debridement
can be very helpful. The end result after the procedure is usually a healed disease with
shortening of limb and moderate to gross restriction of movements. Arthrolysis aims to
achieve the useful range of movements in the cases with gross limitation of movements not
responding to traction and exercises. All pathological and fibrous tissues are excised
carefully without compromising with vascularity of remaining part of the upper end of the
femur. It is wise to leave the posterior capsule undisturbed because it carries vital blood
supply to the femoral head. Posterior capsule is generally not shortened as most of the
patients have flexion deformity. After surgery, skeletal traction is applied, and movements
of the hip are allowed under supervision as soon as patient is able to do.

Advanced arthritis with subluxation/dislocation


Deformities and shortening are another problems to tackle. The different lines of
management of this complex problem could be conservative traction regimen, excision
arthroplasty, arthrodesis, and hip replacement.

19. Answer the following questions:


(a) What are the clinical features of giant cell tumour in the distal radius?
[1.5m]
(b) Draw a diagram to show the radiological features of giant cell tumour of the
distal radius. [1.5m]
(c) What are the principles in its management? [2m]

a) -epiphyseo-metaphyseal, eccentric swelling is seen at the end of long bones


- overlying skin is stretched and shiny but no engorged veins
- skin is warm, tender with bony consistency
b)
c) Tumor removal- extended curettage, en bloc resection
Restore function- allograft, arthrodesis, arthroplasty

20. Describe the complications and management of Acute Osteomyelitis.


A) Acute Osteomyelitis is infection of the bone by pyogenic organisms which is
common in children.
Complications for acute osteomyelitis can be divided into genral and local
 For general complications: In early stage, patient may develop septicemia
and pyaemia.
 For local complications: most of these can be cause by delay in diagnosis
and inadequate treatment.
- Chronic osteomyelitis
- commonest complication for acute osteomyelitis
- hardly diagnosis at early stage
- delay diagnosis lead to sequestrum formation & pent-up pus in
cavities
of bone
- Acute Pyogenic arthritis
- it occur in joints where the metaphysis is intra-articular
- hip & shoulder
- Pathological fracture
- occur in weaken bone by the disease / window made during
surgery
- can be avoided if adequate splinting of limb
- Growth plate disturbance
- damaged leading to complete or partial cessation of growth

B) Management
Investigations:
o Haemoglobin : LOW
o Total WBC : as high as 30,000 with leukocytosis
o ESR: HIGH
o Blood Culture: bacteremia
o Radio CXR:
-Early : normal
- Late : moth-eaten appearance
Periosteal elevation (onion-peel appearance seen in Ewing’s sarcoma)
o CT scan
o MRI

Treatment:
- Antibiotics started as early as possible
- Antibiotics changed later if necessary as per culture and sensitivity
reports
- Immediate drainage of paramount importance: cortical window at
suspected site
- Prolonged antibiotics (Minimum period of 6 weeks with 2 weeks
parenteral antibiotics

21. Types of fracture, complications and principles of management.


Fracture is a complete or incomplete break in the structural continuity of bone
Can be classified into
A) Open/Closed
B) Incomplete/Complete
C) Undisplaced/Displaced
Complete fractures based on line of fracture
- Transverse (Bending)
- Oblique (Angular)
- Spiral fractures (Twist/Rotation)
- Compression
- Comminuted (Crush/Gunshot)
- Segmental

Fracture healing varies depends on


- Closed or open
- Age of patient
- Type of bone involved
- Site of fracture
- Associated injuries
- General systemic condition
Management
Initial : Splint fractures and treat life threatening injuries
Immobilize fracture
Later: Reduce and immobilize
Treat systemic disorders
Reduce fracture can be divided into Open reduction and Closed Reduction
- Open reduction
-when closed reduction fails
- difficulty in controlling fracture
- intra-articular fractures
-unstable fracture
- multiple fracture
-uncooperative patients
- Closed reduction
-all closed fracture
-unless indicated
Complications
- Non-Union & Malunion
- General
- Crush syndrome
-Deep vein Thrombosis
- Tetanus
- Fat embolism
- Shock
- Systemic Inflammatory Response System
- Local
-Early
-Late

23..Answer the following questions:


(a) What are the clinical features of supracondylar fractures of the humerus in a
child? [1.5m]

Symptoms
 pain
 refusal to move the elbow

Physical exam
Inspection
 gross deformity
 swelling
 ecchymosis in ante-cubittal fossa

Motion
 limited active elbow motion

Neurovascular
Evaluate for

AIN neurapraxia
 unable to flex the interphalangeal joint of his thumb and the distal interphalangeal joint of
his index finger (can't make A-OK sign)
Median nerve injury
 loss of sensation over volar index finger
Radial nerve neurapraxia
 inability to extend wrist or MCP joints
 PIP and DIP can still be extended via intrinsic function (ulnar n.)

(b) Draw a diagram to show the types of supracondylar fractures of the humerus.
[1m]

(c) Draw a diagram to show its radiological classification. [1.5m]


Type 1 -Anterior humeral line goes across the capitulum.
Not displaced
Posterior fat pad sign
Type 2- Anterior humeral line goes anterior to capitulum.
Displaced
Posterior cortical contact is present.
Tyope 3- No cortical contact between two fragments.

(d) Enumerate 4 complications of this injury. [1m]

 malunion: resulting in cubitus varus (varus deformity of the elbow, also known as
gunstock deformity)
 ischaemic contracture (Volkmann contracture) due to damage/occlusion to
the brachial artery and resulting in volar compartment syndrome
 damage to the ulnar nerve (most common), median nerve, or radial nerve.
 Infection

24. Answer the following questions:


(a) What are the risk factors in a child for developmental dysplasia of the hip
(DDH)? [1m]

 Breech presentation
 Female sex
 Positive family history
 Firstborn status
 Oligohydramnios.

(b) Enumerate 4 pathological changes in the hip in the presence of DDH. [1m]

o Femoral neck excessively anteverted


o Epiphysis is small and ossifies late
o Acetabulum shallow
o Labrum may be folded into the cavity
(c) Enumerate the clinical features in a newborn child with unilateral DDH. [1m]
o Hip instability (demonstrate by positive Ortolani or Barrow test)
o Assymetric leg creases (inguinal, gluteal, thigh, popliteal)
o Apparent shortening of femur (Galeazzi )

(d) What are the principles in its management in a new born child with DDH? [2,]

Dislocated or unstable hips in newborn infants can usually be held in place by a brace or
harness that holds the legs in a position while the socket and ligaments become more stable.

There are a wide variety of holding devices available, but the most common ones are the
Pavlik Harness, or various types of devices called fixed abduction braces.

Most doctors recommend full-time wear for 6-12 weeks but some doctors allow removal
for bathing and diaper changes as long as the legs are kept apart to keep the hips pointed at
the socket. After the hips become stable, the brace is worn part time, usually at night, for
another 4-6 weeks.

25. Complications of Traumatic Paraplegia and describe their management


Complication
- Unable to control bladder and bowel movement
- Loss of skin sensation
- Loss of circulatory control (increase the risk of developing blood clots, such as deep
vein thrombosis or a pulmonary embolus)
- Difficulty of breathing and coughing if the abdominal and chest muscles are affected.
- Loss of muscle tone (uncontrolled tightening or motion in the muscles (spasticity) or
soft and limp muscles lacking muscle tone (flaccidity)
- Weight loss and muscle atrophy
- Affects sexual health (Men: changes in erection and ejaculation; women: changes in
lubrication)
- Pain due to overuse of particular muscle groups
- Depression
Management
First, immediate immobilization of the spine as gently as possible using a rigid neck
collar and a rigid carrying board can be done to transport the patient to the hospital.

When reaching the emergency room, the doctor must first maintain the ability of the
patient to breathe (intubation). Fluids like crystalloid and colloid can also be given to
prevent haemorrhagic shock in severe trauma case. Then, immobilization of neck must be
done to prevent further spinal cord damage. Avoiding possible complications, such as
stool or urine retention, respiratory or cardiovascular difficulty, and formation of deep
vein blood clots in the extremities is also very important.

Medication such as Intravenous (IV) methylprednisolone (A-Methapred, Solu-Medrol)


can be use for acute spinal cord injury (moniter for side effect like blood clots and
pneumonia).

Immobilization can be done by traction to stabilize the spine and to bring the spine into
proper alignment or both. In some cases, a rigid neck collar may work. A special bed also
may help to immobilize the body.

Surgery is often necessary to remove fragments of bones, foreign objects, herniated disks
or fractured vertebrae that appear to be compressing the spine. Surgery may also be
needed to stabilize the spine to prevent future pain or deformity.

26. Aetiology, clinical features, management and complications of Supracondylar


fractures of humerus. Supracondylar fracture of humerus in a young
child.(*******)
Etiology
Supercondylar fracture is the most common fracture in children, usually occurs in
children from 5 to 7 years old. Supracondylar fracture is the visible fracture of the distal
humerus. It occurs when an axial load on the elbow, with the olecranon acting as a wedge
splitting the medial and lateral columns of the distal humerus. The fracture pattern is
related to the degree of elbow flexion and the direction and magnitude of the force
applied. When trauma occurs, the hemathrosis will raises the fat pads away from the
humerus, the powerful triceps muscle then posteriorly displaces the ulna, taking the
capitulum with it. The capitulum therefore lies well behind the anterior humerus line (at
least 1/3 of the capitulum should lie in front of the anterior humerus line.

Clinical Feature
- A snap or pop at time of injury
- Sudden intense pain in the elbow and forearm.
- Painful and swelling elbow that the patient unable to move or straighten the arm.
- Numbness in the hand
- Open wounds may present

Management
For mild fracture, a cast or splint can be used to immobilize the joint and allow it to heal
naturally. A splint and a cast also can be used together, which the splint is used to allow
the swelling to go down, followed by a full cast. Closed reduction also can be done to set
the bones back into place before applying the splint.
For severe fracture, closed reduction with percutaneous pinning can be done. The doctor
will insert pins through the skin to rejoin the fractured parts of the bone. Then a splint is
applied for the first week and then replaced by a cast. Open reduction with internal
fixation also can be done if there is damage to the nerves or blood vessels.
Complications
- Infection
- Reduced stiffness of the elbow
- Ulnar neuropathy
- Nonunion or malunion of the distal humerus and olecranon osteotomy
- Nonunion of the olecranon osteotomy
- Hardware irritation
- Iatrogenic nerve injuries
- Post operative osteoporosis
- Overall functional disability

27. Aetiology, pathology, clinical features and management of acute haematogenous


osteomyelitis (**) of lower end of femur. Outline differential diagnosis and
complications.
Aetiology
Acute hematogenous osteomyelitis typically arises in metaphysis of long tubular bone
and is caused by bacteraemia.Bacteriological seeding of bone generally is associated with
other factors such as localized trauma,malnutrition,impaired immune system, and chronic
illness.
In infants: Staph. Aureus, Steptococcus agalactiae, E.coli
Children over 1 year old: Staph.a, H.influenza, Steptococcus pyogenes
Staphylococcus aureus is the most commonly isolated pathogen.
Pathology
Terminal branches of metaphyseal arteries form loops at growth plate and enter irregular
afferent venous sinusoids. Blood flow slowed and turbulent, predisposing to bacteria
seeding. In addition, lining cells have little or no phagocytic activity and abscess may
form.Abscess limited by growth plate, spreads transversely along Volkmann canal and
elevate periosteum.
Clinical Presentation
-Pain of affected area and loss of function(movement restricted)
-Malaise
-Presenting with fever
-Tachycardia
-Localized redness, pain, warm, edema is a late sign and indicate pus collection in soft
tissue.
-As the lower extremity is more commonly affected, a common presentation is a child
with a limp.
Treatment:
 Antibiotic: Empiric therapy in children should adequately cover S. aureus based
on local susceptibility patterns.
o First Generation Cephalosporin (e.g. cefazolin)
o Anti-staph Penicillin (nafcillin, oxacillin, etc)
o Clindamycin (if suspecting MRSA and local resistance to
clindamycin is low)
o Vancomycin
o Linezolid
Choice between IV and PO antibiotics
o Historically, AHOM treated with long courses of IV antibiotics

o Recent data suggests that a 2-4 day course of IV antibiotics,


followed by oral antibiotics is as efficacious as IV therapy alone in
uncomplicated cases.
 Surgical : For abscess collection
Patient is not responding to appropriate antibiotic therapy after a
negative bone aspiration.

Differential Diagnosis
-Acute osteomyelitis
-Acute septic arthritis
-Acute rheumatic arthritis
-Cellulitis
Complications
-Chronic osteomyelitis
-Growth disturbance
-Septic arthritis

28. Aetiology, clinical features, management and complications of intracapsular


fractures of neck of femur.
Aetiology

The commonest cause in the elderly is generally a fall onto the side of the fracture.The
primary risk factors is osteoporosis but also other age-related issues which might make a
frail patient likely to fall (i.e. poor vision, poor proprioception, arthritis, dementia).

Clinical features

o Fall followed by pain in the groin with referred pain to the thigh
o Limited ability to weight bear
o Limited range of movement (particularly straight leg raise)
o External rotation with shortening of the limb length in displaced fractures

Management
This depends on the performance status of the patient as well as the displacement of the
fracture

o Non-displaced fractures (Garden I+II)

-Relatively young patients (either chronologically or more important, physiologically)


should have urgent internal fixation via 3 or 4 parallel partially threaded cancellous
screws.

o Displaced fractures (Garden III+IV), or even non-displaced fractures in the context of


patients unlikely to tolerate non-weight bearing.

-Replacement of the femoral head to obviate the risks of avascular necrosis.This is


most often via a hemiarthroplasty. Younger, fitter patients are being offer a primary
total hip replacement (THR) in centres able to provide the service.

Complications

Avascular necrosis

Mal-union or non-union of the fracture

Infection

Venous thromboembolic disease

29. Aetiology, pathology, clinical features, radiological findings of Perthes Disease.


Mention differential diagnosis. (*)
This is an osteochondritis of the epiphysis of the femoral head. In this disease, the
femoral head becomes partly or wholly avascular and deformed.
Aetiology : The cause is not definitely known, but it is supposed to be due to recurrent
episodes of ischaemia of the head in the susceptible age group, probably precipitated by
episodes of synovitis.
Pathology : the disease progresses in three ill-defined stages: (i)stage of synovitis; (ii)
stage of trabecular necrosis; and (iii) stage of healing. The disease occurs commonly in
boys in the age group of 5-10 years.
Clinical Features : The child presents with pain in the hip, often radiating to the knee.
There may be limping or hip stiffness. On examination, findings may be minimal –
sometimes the only findings being a limitation of abduction and internal rotation and
shortening.
Radiological examination : Reveals collapse and sclerosis of the epiphysis of the femoral
head. Hip joint space is increased. In fact, the contrast between the paucity of symptoms
and signs in the presence of gross X-ray changes is striking. Bone scan may show a
decreased uptake by the head of the femur.
30. What is a pathological fracture? Briefly outline how to investigate and manage
an elderly patient admitted with pathological fracture of shaft of femur.
A fracture is termed pathological when it occurs in a bone made weak by some disease .
Often, the bone breaks as a result of a trivial trauma, or even spontaneously.

Investigation : A fracture sustained without a significant trauma should arouse suspicion


of a pathological fracture. Often the patient, when directly questioned, admits to having
suffered from some discomfort in the region of the affected bone for some time before
the fracture. The patient may be a diagnosed case of a disease known to produce
pathological fractures (e.g., a known case of malignancy), thus making the diagnosis of a
pathological fracture simple. At times, the patient may present with a pathological
fracture, the cause of which is determined only after a detailed work up.

Management : Treatment of a pathological fracture consists of: (i) detecting the


underlying cause of the fracture; and (ii) making an assessment of the capacity of the
fracture to unite, based on the nature of the underlying disease. imperfecta or
osteoporosis is expected to unite with conventional methods of treatment. A fracture at
the site of a bone cyst or a benign tumour will also generally unite, but the union may be
delayed. Fractures occurring in osteomyelitic bones often take a long time, and
sometimes fail to unite despite best efforts. Fractures through metastatic bone lesions
often do not unite at all, though the union may occur if the malignancy has been brought
under control with chemotherapy or radiotherapy. With the availability of facilities for
internal fixation, more and more pathological fractures are now treated operatively with
an aim to: (i) enhance the process of union by bone grafting (e.g. in bone cyst or benign
tumour); or (ii) mobilise the patient by surgical stabilisation of the fracture. Achieving
stable fixation in these fractures is difficult because of the bone defect caused by the
underlying pathology. The defect may have to be filled using bone grafts or bone cement.

31. Common deformities of knee region seen in children and adolescents. Discuss
aetiology and management.
a) Genu Valgum
Etiology
It is well recognized that toddlers aged 2-6 years may have physiologic genu valgum. For
this age group, typical features include ligamentous laxity, symmetry, and lack of pain or
functional limitations. Despite the sometimes-impressive deformities, no treatment is
warranted for this self-limiting condition. Bracing is meddlesome and expensive, and
shoe modifications are unwarranted. The natural history of this condition is benign;
therefore, parents simply need to be educated as to what to expect and when. Annual
follow-up until resolution may help to assuage their fears.
In contrast, adolescent idiopathic genu valgum is not benign or self-limiting. Teenagers
may present with a circumduction gait, anterior knee pain, and,
occasionally, patellofemoral instability. The natural history of this condition may
culminate in premature degenerative changes in the patellofemoral joint and in the lateral
compartment of the knee.
Various other conditions, including postaxial limb deficiencies, genetic disorders such as
Down syndrome, hereditary multiple exostoses, neurofibromatosis, and vitamin D–
resistant rickets may cause persistent and symptomatic genu valgum. Some of these
conditions require team management with other health care providers; however, surgical
intervention is still likely to be necessary to correct the malalignment of the knees.

Medical Therapy
For the child with specific and identifiable bone dysplasia, medical treatment may have
an important role, influencing the outcome. For example, the child with vitamin D–
resistant rickets should be on appropriate medication to optimize bone formation and
mineralization. Likewise, children with osteogenesis imperfecta may benefit from
treatment with bisphosphonates to increase bone density and decrease the risk of
fractures.
Recognizing the need for holistic care, even optimal medical management does not
correct preexisting genu valgum. However, treatment may slow the progression of the
condition and prevent recurrence. Bracing and physical therapy may provide a temporary
reprieve of symptoms, but they do not afford long-term symptomatic relief.

Surgical Options
Stapling has waned in popularity since its introduction, having been supplanted by the
tension band plate concept. Reference works typically dismiss it as a historical procedure,
citing unpredictability and the fear of permanent physeal arrest as results of stapling.
Although stapling can work well, occasional breakage or migration of staples can
necessitate revision of hardware or premature abandonment of this method of treatment.
(See the image below.)
Heretofore, stapling was a viable option.
This outpatient procedure permitted
simultaneous and multiple deformity
correction, without casts or delayed
weightbearing. However, the concept of
compressing and overpowering the
physes has the drawbacks of slower
correction because the fulcrum is within
the physis. Provided the rigid staples did
not dislodge or fatigue, satisfactory
correction could be realized. If the
hardware failed prematurely, the
correction was either abandoned or the
hardware exchanged. Compared with
osteotomies, it was a risk worth taking,
that is, until the advent of a better option.

Some surgeons have reverted to osteotomy of the femur and/or tibia-fibula as the
definitive means of addressing genu valgum. However, this is a very invasive method
fraught with potential complications, including malunion, delayed healing, infection,
neurovascular compromise, and compartment syndrome. Further complicating the
picture, these deformities are often bilateral, requiring a staged correction. The aggregate
hospitalization, recovery time, costs, and risks make osteotomy a last resort for angular
corrections (unless the physis has already closed).
Percutaneous drilling or curettage of a portion of the physis yields only a small scar and
no implant is required. However, this is a permanent, irreversible technique. Therefore,
its use is necessarily restricted to adolescent patients and is predicated upon precise
timing of intervention, requiring close follow-up to avoid undercorrection or (worse yet)
overcorrection.
Some authorities advocate using percutaneous epiphyseal transcutaneous screws as a
means of achieving angular correction. [6, 7] Although this is performed through a small
incision, the physis is violated, and the potential exists for the formation of an unwanted
physeal bar, with its sequelae. To date, the potential for reversing the procedure has not
been documented in younger children; therefore, the only reported cases have been in
adolescents.
By comparison, guided growth, using a nonlocking two-hole plate and screws, is a
reversible and minimally invasive outpatient procedure, allowing multiple and bilateral
simultaneous deformity correction. A single implant is used per physis (see the images
below); this serves as a tension band, allowing gradual correction with growth. Because
the focal hinge of correction (CORA) is at or near the level of deformity, compensatory
and unnecessary translational deformities are avoided. [8,9]

The application of a single 8-plate per


physis permits the same correction as
stapling, without the potential drawbacks
of implant migration or fatigue failure.
Based on the principle of facilitating
rather than compressing the physes, the
correction occurs more rapidly and
rebound growth, though possible, may
be less frequent. When the mechanical
axis has been restored to neutral, the
plates (or metaphyseal screws) are
removed (and replaced as necessary if
recurrent deformity ensues).
presented with activity-related anterior
knee pain, circumduction gait, and
difficulty with running and sports. His
symptoms had been progressive over a
period of 18 months despite
nonoperative measures including
physical therapy, activity restrictions,
and nonsteroidal anti-inflammatory drug
therapy.

This 14-year-old boy, weighing 132 kg,

Nine months following the insertion of


8-plates in the distal femora (1 per knee),
the mechanical axis is approaching
neutral and his symptoms abated. The
plates were removed 2 months later,
allowing for full correction of his valgus
deformities. He has not had recurrence.

The previous empirical constraints related to the indications for instrumented


hemiepiphysiodesis, including appropriate age group and the etiology of deformity, have
been challenged successfully with this technique, and results have been consistently
good. During the past decade, in a personal series of more than 1000 patients ranging in
age from 19 months to 18 years, some of whom had pan-genu deformities, the senior
author has not had a permanent physeal closure, nor have any been reported in the
literature.
Guided growth has emerged as the treatment of choice in the growing child; osteotomy
should be reserved as a salvage option (or for mature patients). Despite the age of the
child or the etiology of the valgus, even children with "sick physes" may be well served
by the application of an extraperiosteal two-hole plate at the apex (or apices) of the
deformity. The ensuing growth should correct the deformity within an average of 12
months. This is documented with quarterly follow-up evaluations, including full-length
radiographs with the legs straight.
When the mechanical axis has been restored to neutral, the implants are removed. Growth
should be monitored because if the valgus recurs, guided growth may have to be
repeated. The goal is to correct the deformity, which alleviates the pain and gait
disturbance and protects the knee throughout the growing years. If this requires repeated,
yet minor, intervention, the benefits still outweigh the cost and risks of (sometimes)
repeated osteotomies. If recurrence is anticipated, an option is to remove the metaphyseal
screw percutaneously, monitor subsequent growth, and insert another screw as needed.

b) Genu Varum.
The recognized etiologies for genu varum include the following:
 Tibia vara ( Blount disease) – Infantile, juvenile, adolescent (see the first image
below)

 Rickets – Hypophosphatemic, nutritional, renal disease (see the second image


below)

 Skeletal dysplasias – Achondroplasia, pseudoachondroplasia, multiple epiphyseal


dysplasia, metaphyseal dysplasia

 Celiac sprue and other digestive disorders


Tibia vara (Blount disease) is growth
disturbance of proximal medial tibia that
can present any time from infancy to
adolescence. Natural history is one of
inexorable progression, premature
closure of upper medial tibial physis,
lateral thrust, ligamentous laxity, and,
ultimately, joint instability and
degeneration. At age 5, guided growth
would have been sufficient. After
physeal closure, complex osteotomies
are required.
Hypophosphatemic rickets is disturbance
in vitamin D metabolism that weakens
physes through delayed ossification.
Consequent deformities may progress
despite careful medical management and
bracing. Deformities are typically
bilateral, involving both femur and tibia.

Medical Therapy
For physiologic genu varum, parental reassurance is required, but no treatment is
necessary; spontaneous resolution by the age of 2 years is the rule. In borderline cases,
continued follow-up is warranted.
To resolve pathologic genu varum, some have relied on so-called Forrest Gump above-
the-knee braces. However there are no controlled, randomized trials that support the
efficacy of such treatment. Furthermore, the laxity of pediatric collateral ligaments may
militate against such a management strategy because the force applied by the braces may
be expended upon the ligaments. The cost and restrictive nature of such braces further
diminish compliance.
In defined metabolic conditions, such as rickets, medical management is paramount for
success. There may be other appropriate measures that should be taken, such as dietary
management for celiac sprue, administration of bisphosphonates in select cases of
osteopenia, and gene therapy for collagen storage disorders.

c) Genu recurvatum
Etiology

The injuries that resulted in genu recurvatum are usually caused by an unexpected impact
to the extended knee following an injury to some structures of the knee or just the
posterior aspect of the knee structures. Other causes involve:

 A connective tissue disorder


 Looseness of the knee ligaments
 An inherited problem or birth defect
 Joints of the knee are not stable due to ligaments
 Femur and tibia is not properly aligned
 Postural habits
 Some medical diseases like multiple sclerosis, cerebral palsy, or muscular
dystrophy
 Length of the lower limb is not the same
 Unusual position of the ankle and foot when walking

Treament

Physical Therapy
At the start, the physician might recommend physical therapy to enhance the strength of
the quads to compensate for the back knee. The treatment involves gait training to help
the person concentrate on the correct sequencing and keeping control on the limb.
Another one is the proprioceptive training which can improve a person’s balance,
coordination, and agility and prevent other injuries in the future.

Orthoses
This gives a favorable support to the knee since it can control the abnormal bending of
the knee-joints and stabilizes the leg.

d) Triple deformities
Etiology

Is described as inflexed position of the knee, the tibia subluxate sposteriorly &laterally &
also rotates laterally over the femoral condyle. Gradually leg also goes in valgus. Thus
though it has been identified as triple subluxation, actually it is a“quadruple deformity
complex.”

Main causes are tuberculosis, rheumatoid arthritis, postural contracture.

The disease may begin either in bone, usually in the femoral or tibial epiphysis or in the
patella or synovial membrane, later being the most frequent site.

The synovial membrane is thickened, grey &translucent, & in places gelatinous or even
caseous.

Fluid is present in varying amount, &adhesions form so that the outlying synovial
pockets become loculated.

Granulations spread under & over the cartilage, which, being eroded by pressure &
friction, may become detached, leaving the bone exposed.

At the same time softening & stretching of the ligaments tend to produce subluxation of
the tibia, which slips backwards& rotates laterally(triple deformity).

Inflammation takes place leading to thickening of the joint, so called spindle-shaped


tumor, known as ‘white swelling’ is formed.

Treatment
Conservative treatment:

-skin traction

-Thomas splint with knee flexion piece applied, which allows the pull to be made inthe
line of deformity, & as treatment progresses, it can be adjusted daily until full extension
is obtained.

-Care must be taken to avoid backward rotational displacement of tibia & is achieved by
reverse dynamic sling.

-AKT should be started

Operative treatment:

-Extra-articular abscess should be drain out

-Synovectomy, in those whose knees remain warm & swollen after conservative
treatment.

-Arthrodesis, where there is marked destruction of joint.

e) Flexion deformity
Etiology
A number of relatively common conditions, especially cerebral palsy and spina bifida,
may lead to progressive FKFD, despite appropriate physical therapy and bracing.
Congenital FKFD, with or without fixed lateral dislocation of the patella, may be evident
on perinatal ultrasonography.
Treatment
Medical Therapy

Medical therapy for FKFD usually consists of onabotulinumtoxinA injections in the


hamstrings or baclofen administered orally or through an intrathecal pump (for cerebral
palsy). This is only useful for the dynamic component of crouch gait; it may be an
adjunct to osteotomy or guided growth. The patient is often working with a physical
therapist on hamstring stretching, quadriceps strengthening, and gait training. This
approach is suitable for younger patients, but after the age of 10 years, deformities are
likely to progress despite concerted efforts to the contrary.

Surgical Therapy

Posterior capsulotomy/hamstring recession

This relatively invasive soft tissue procedure poses some risks to the posterior
neurovascular structures and requires immobilization with braces, casts, or frames. (See
the image below.)

For flexion contracture, spasticity


management
(onabotulinomtoxinA/phenol/baclofen)
or hamstring recession may offer some
improvement, but these measures cannot
overcome fixed knee flexion deformity.

Osteotomy

Supracondylar extension osteotomy of the femora has a long track record and is the
default mode for many surgeons. Unfortunately, there are associated drawbacks, not the
least of which is recurrence with growth, thus mitigating the temporary benefit of this
maximally invasive treatment. (See the images below.) The varied techniques, tricks,
results, and complications have been well described in standard textbooks and journals.
Starting at age 4 years, this patient
subsequently underwent bilateral
extension osteotomies 4 times, with
recurrence each time as expected.
Perhaps this sequence could have been
abbreviated with guided growth, which,
even if repeated, requires no casts or
delay in weight bearing.

presented with a congenital knee flexion


deformity. His only prior surgery was a
Symes disarticulation for fibular absence
and a rigid teratologic foot deformity.
He was ambulatory in a prosthesis.

Part 1 of 5. This 5-year-old boy


Part 2 of 5. This patient underwent a
supracondylar extension osteotomy of
the femur.

Part 3 of 5. Because of a relatively rapid


recurrence of fixed knee flexion
deformity, this patient underwent
anterior stapling of the femur;
unfortunately, the staples migrated, but
the physis is still open.
Part 4 of 5. The staples in this patient
were retrieved and replaced with a pair
of eight-plates.

clock, perhaps guided growth would


have been sufficient to correct the
problem in this patient, without an
osteotomy or cast. The effective gain in
limb length would occur gradually,
without risk to the neurovascular
structures.

Part 5 of 5. If we could turn back the


Frame distraction

With or without soft tissue release, some authors favor frame distraction as a means of
gradual correction of FKFD. However, the bilateral nature of these problems makes this
method relatively expensive and unwieldy. Furthermore, even with protracted bracing,
recurrence is relatively common.
This girl born with a teratologic knee
flexion deformity and absent quadriceps
had previous posterior capsulotomy,
supracondylar osteotomy, and attempted
stapling. Subsequently, she had a spatial
frame applied to gradually extend the
ankylosed knee; however, she fell and
sustained a Salter I fracture of the
proximal tibia.

Guided growth

Guided growth is a newer approach that consists of anterior hemiepiphysiodesis of the


distal femora. Stapes were originally used, but it became apparent that some children
were relatively small for the Blount staples. These rigid devices would occasionally
migrate or permit relatively slow correction.

Using a pair of eight-plates as an alternative has resolved the problem of hardware


migration and enabled more rapid correction. The titanium plates, though intracapsular,
are nonarticular, being placed medial and lateral to the patellofemoral sulcus. Thus, they
are well tolerated, even by young children. (See the images below.)
the fixed knee flexion deformity at or
close to the level of the CORA (center of
rotational axis of deformity). This is
efficient and prevents the need for
translocation, such as is required in an
osteotomy. The gradual correction poses
no risk to the neurovascular structures.

Guided growth permits one to address


With the C-arm in the lateral, horizontal
position, the physis is localized. A Keith
needle is placed in the physis, and two
1.6-mm guide pins are inserted: one
medial and one lateral to the sulcus. The
cannulated 4.5-mm screws are then
inserted. They need not be parallel, but
they should not transgress the physis,
joint, or posterior cortex.
eight-plate is placed on either side of the
patellofemoral sulcus, through a small
arthrotomy. Though intracapsular, the
plates are nonarticular; synovitis has not
been observed.

For fixed knee flexion deformity, an


bracing may be unnecessary, provided
the quadriceps are sufficiently strong.

The efficacy of floor reaction braces is


compromised in the presence of fixed
knee flexion deformity. However, they
may be continued following guided
growth, pending correction, whereupon
If one elects to employ guided growth, it is important to ascertain whether the distal femoral physes
are open and whether there is, ideally, 12 months or more of predicted growth remaining. The
decision to undertake concomitant multilevel reconstructive procedures, including hamstring
recession for dynamic contracture, is left to the discretion of the surgeon.

Note, however, that as FKFD gradually corrects, there may be beneficial effects upon the hip, spine,
and ankle. Therefore, it may be wise to await full knee extension and address residual deformities at
the time of eight-plate removal.

The key aspects of the procedure may be summarized as follows:

 Supine position with knee flexed on bolster

 Tourniquet control

 Image intensifier: parked in cross-table, horizontal position

 Two incisions (3 cm long), medial and lateral to the patella

 Open capsule and synovium

 Keith or similar needle (sequentially) placed into the anteromedial and anterolateral physis

 Application of an eight-plate (eg, Orthofix), usually 16 mm

 Introduction of 1.6 guide pins, first epiphyseal and then metaphyseal - Pins need not be
parallel but should avoid the physis and joint

 Predrilling of cortex (to a depth of 5 mm)

 Insertion of the 4.5-mm titanium, self-tapping, cannulated screws (24 or 32 mm in size)

 Soft dressing

32. 40-year-old man falls from the roof hitting the back of his neck against the sunshade and
landed on the feet over the ground and sustained multiple injuries. Discuss the injuries
and management.
The Cervical Spine

The cervical spine is located at the very top of the spinal column. The seven vertebral levels within
this region, which are classified as C1-C7 from the top down, form the human neck. There is an
additional cervical-level injury known as a C8 injury which relates to damage to the spinal cord root
that exits the spinal column between vertebrae C7 and T1.

The spinal cord running through the cervical region of the spine is identified by the level of the
vertebra in which it’s contained. Cervical spinal cord injuries are the most severe of all spinal cord
injuries and may affect one or both sides of the body.

The higher up in the spine that the injury occurs, the more severe the potential outcome. Some
cervical spinal cord injuries are severe enough to result in death. Injuries to C1 and C2 are very rare
and most injuries to the cervical spinal column occur near the C4 / C5 levels. While no two spinal
cord injuries are the same, early treatment is critical to the long-term prognosis of any injury to the
cervical spinal column.

Cervical Spinal Cord Injury Levels

High-Cervical Nerves (C1-C4)

The C1 and C2 vertebrae form the top of the spine (neck) at the base of the skull. These bones are
named atlas and axis respectively and support the pivot motion of the neck. Injuries to the spinal
cord at the C1 & C2 levels are rare, extremely severe, and most often fatal. Atlas and axis are
followed by C3 and C4 to form the high cervical vertebrae. If not fatal, complete damage to the
spinal cord or nerves corresponding to any of the high-cervical vertebral levels most often results
in full paralysis, or quadriplegia. A survivor may not be able to breathe on their own and will likely
require 24-hour care for the rest of their life.

Low-Cervical Nerves (C5-C8)

An individual with an injury to the cervical spinal cord at the C5 level or below has a greater chance
of retaining some motor and sensory function than a patient who experiences an injury to the C1-C4
levels. Though damage to the spinal cord at any portion of the neck has the potential to result in
full paralysis of each of the four limbs, survivors of C5-C8 injuries may be able to breathe on their
own and speak normally.

Common Cervical Injury Symptoms

Patients with cervical spinal cord injuries will likely experience to some degree:

 Inability to breathe on one's own without assistance (C1-C4)

 Impaired ability or inability to speak (C1-C4)


 Numbness, tingling, or loss of feeling below the level of the injury

 Paralysis in the legs, torso, and arms

 Inability to control bladder and bowel function

 Inability to groom or dress oneself

Treatment & Recovery

The unfortunate truth of spinal cord injuries is that there is no way to reverse damage to the spinal
cord at any level. With cervical spinal cord injuries being the most severe of all spinal cord injuries,
patients will have a long road to rehabilitation ahead of them.

Early treatment may include:

 Stabilizing and securing the neck to prevent further damage.

 Regulation of breathing and respiratory function.

 Prevention of complications including blood clots and stool or urine retention.

Long-term treatment and rehabilitation may include:

 Surgery to reduce pressure on the spinal cord.

 Steroid injections and pain regimens to reduce any discomfort and inflammation from the
injury.

 Physical therapy to aid in regaining function of the affected parts of the body and to
maintain the function in areas which were not affected by the injury.

 Stem cell injections are a new tool to aid in the recovery of the spinal cord. The use of stem
cells has not been widely adopted, however, promising clinical data has been reported.

33. Classify type of epiphyseal injuries. Aetiology, clinical signs and symptoms, corn
plications, and management of fracture of medial condyle of humerus in 8 year old child.
Salter and Harris Classification:
Type 1: A transverse fracture through the hypertrophic or calcified zone of the plate. Even if
the fracture is quite alarming displaced, the growing zone of the physis is usually not injured
and growth disturbance is uncommon.
Type 2: Essentially similar to type 1 but towards the edge the fracture deviates away from
the physis and splits off a triangular metaphyseal fragment of bone. (Thurston-Holland
fragment)
Type 3: A fracture that splits the epiphysis and then veers off transversely to one or the other
side, through the hypertrophic layer of the physis. (Growth disturbance may occur)
Type 4: As with type 3, the fracture splits the epiphysis, but it extends into the metaphysis.
(Liable to displacement and a consequent misfit resulting in asymmetrical growth)
Type 5: A longitudinal compression injury of the physis. (Growth arrest)

Fracture of medial condyle of humerus:


I. Mechanism of injury: fall from height, either from direct blow to the point of the
elbow or a landing on the outstretched hand with the elbow forced into valgus; in
the latter case it would be an avulsion injury.
II. Clinical features: considerable pain and swelling, in older children the metaphyseal
component if usually easily visualized on x-ray. In younger children, much of the
medial condylar epiphysis is cartilaginous and therefore, not visible on x-ray.
Arthrogram may be helpful.
III. Complications: EARLY- early reduction of both dislocation and the fracture, if
necessary by open operation and pinning is important. Ulnar nerve damage is
uncommon but recovery is usual unless the nerve is left kinked in the joint. LATE-
Stiffness of elbow is common and extension often limited for months.
IV. Treatment: Undisplaced fractures are treated by splintage, x-rays are repeated until
fractures are healed, so as to ensure that is does not become displaced. Displaced
fractures are treated either by closed reduction or by open reduction and fixation
with pins. Post-op management.

34. Pathology, clinical features and differential diagnosis of osteosarcoma (*). treatment of
osteosarcoma of upper tibia in a 16 year old boy.
I. Pathology: The tumor is usually situated in the metaphysis of a long bone, where it destroys
and replaces normal bone. Areas of bone loss and cavitation alternate with dense patches of
abnormal new bone. The tumor extends within the medulla and across the physeal plate.
There may be obvious spread into the soft tissues with ossification at the periosteal margins
and streaks of new bone extending into the extra osseous mass. The histological appearances
show considerable variation: some areas may have the characteristic spindle cells with a
pink-staining osteoid matrix; others may contain cartilage cells or fibroblastic tissue with
little or no osteoid. Several samples may have to be examined; pathologists are reluctant to
commit themselves to the diagnosis unless they see evidence of osteoid formation.
II. Clinical features: Pain is usually the first symptom; it is constant, worse at night and
gradually increases in severity. Sometimes the patient presents with a lump. Pathological
fracture is rare. On examination there may be little to find except local tenderness. In later
cases there is a palpable mass and the overlying tissues may appear swollen and inflamed.
The ESR is usually raised and there may be an increase in serum alkaline phosphatase.
III. Differential diagnosis: osteomyelitis, solitary enostosis, brodie’s abscess, osteosarcoma,
osteoblastoma, ewing’s sarcoma
IV. Treatment: The appalling prognosis that formerly attended this tumour has markedly
improved, partly as a result of better diagnostic and staging procedures, and possibly
because the average age of the patients has increased, but mainly because of advances in
chemotherapy to control metastatic spread. However, it is still important to eradicate the
primary lesion completely; the mortality rate after local recurrence is far worse than
following effective ablation at the first encounter. The principles of treatment are outlined
on page 192. After clinical assessment and advanced imaging, the patient is admitted to a
special centre for biopsy. The lesion will probably be graded IIA or IIB. Multi-agent
neoadjuvant chemotherapy is given for 8–12 weeks and then, provided the tumor is
resectable and there are no skip lesions, a wide resection is carried out. Depending on the
site of the tumour, preparations would have been made to replace that segment of bone with
either a large bone graft or a custom made implant; in some cases an amputation may be
more appropriate. The pathological specimen is examined to assess the response to
preoperative chemotherapy. If tumour necrosis is marked (more than 90 per cent),
chemotherapy is continued for another 6–12 months; if the response is poor, a different
chemotherapeutic regime is substituted. Pulmonary metastases, especially if they are small
and peripherally situated, may be completely resected with a wedge of lung tissue.

35. Classify bone tumours(***). Clinical features, investigations and treatment of osteogenic
sarcoma/ osteosarcoma of lower end of femur(*). Note on chemotherapy and
radiotherapy. Radiological differences between osteosarcoma and osteoclastoma.

Osteosarcoma of lower end of femur


Clincal features :
• Pain is usually the first symptom (it is constant, worse at night and gradually increases in
severity).
• Sometimes the patient presents with a lump.
• Pathological fracture is rare.
• On examination there may be little to find except local tenderness.
• In later cases there is a palpable mass and the overlying tissues may appear swollen and
inflamed.
Investigations :
1)X-rays
 The x-ray appearances are variable:
• hazy osteolytic areas may alternate with unusually dense osteoblastic areas.
• The endosteal margin is poorly defined.
• Often the cortex is breached and the tumour extends into the adjacent tissues.When this
happens, streaks of newbone appear, radiating outwards from the cortex – theso-called
‘sunburst’ effect.
• Where the tumour emerges from the cortex, reactive new bone forms at the angles of
periosteal elevation (Codman’s triangle).
• While both the sunburst appearance and Codman’s
triangle are typical of osteosarcoma, they may occasionally
be seen in other rapidly growing tumours.

a) The metaphyseal site; increased density, cortical erosion and periosteal reaction are
characteristic.
(b) Sunray spicules and Codman’s triangle; (c) the same patient after radiotherapy.
(d) A predominantly osteolytic tumour.
• In most cases the diagnosis can be made with confidence on the x-ray appearances.
However, atypical lesions can cause confusion.
• Conditions to be excluded are post-traumatic swellings, infection, stress fracture and the
more aggressive ‘cystic’ lesions.
• Other imaging studies are essential for staging purposes.
 Radioisotope scans may show up skip lesions, but a negative scan does not exclude them.
 CT and MRI reliably show the extent of the tumour.
 Chest xrays are done routinely, but pulmonary CT is a much more sensitive detector of lung
metastases.
• About 10 per cent of patients have pulmonary metastases by the time they are first seen.
• A biopsy should always be carried out before commencing treatment; it must be carefully
planned to allow for complete removal of the tract when the tumour is excised.
Treatment :
• The appalling prognosis that formerly attended this tumour has markedly improved, partly
as a result of better diagnostic and staging procedures, and possibly because the average age
of the patients has increased,
• Mainly because of advances in chemotherapy to control metastatic spread.
• However, it is still important to eradicate the primary lesion completely. The mortality rate
after local recurrence is far worse than following effective ablation at the first encounter.
• After clinical assessment and advanced imaging, the patient is admitted to a special centre
for biopsy.
• The lesion will probably be graded IIA or IIB. Multiagent neoadjuvant chemotherapy is
given for 8–12 weeks and then, provided the tumour is resectable and there are no skip
lesions, a wide resection is carried out.
• Depending on the site of the tumour, preparations would have been made to replace that
segment of bone with either a large bone graft or a custommade implant.
• In some cases an amputation may be more appropriate.
• The pathological specimen is examined to assess the response to preoperative
chemotherapy.
• If tumour necrosis is marked (more than 90 per cent),chemotherapy is continued for another
6–12 months;if the response is poor, a different chemotherapeutic regime is substituted.
• Pulmonary metastases, especially if they are small and peripherally situated, may be
completely resected with a wedge of lung tissue.
Radiological differences between osteosarcoma and osteoclastom
36. Classify hip dislocations. Causes of dislocation of hip. Describe mechanism of injury, signs,
treatment and complications of traumatic posterior dislocation of the hip joint. (*)
- Hip dislocations are classified according to the direction of the femoral head displacement:
posterior (by far the commonest variety), anterior and central (a comminuted or displaced
fracture of the acetabulum).

Causes of dislocation of hip


Posterior dislocation :
Mechanism of injury
- usually occurring in a road accident when someone seated in a truck or car is thrown
forward, striking the knee against the dashboard. The femur is thrust upwards and the
femoral head is forced out of its socket; often a piece of bone at the back of the acetabulum
(usually the posterior wall) is sheared off, making it a fracture-dislocation. Seat-belt
restraints can reduce the number of posterior hip dislocations.
Clinical features
- the leg is short and lies adducted, internally rotated and slightly flexed. However, if one of
the long bones is fractured – usually the femur – the injury can easily be missed as the limb
can adopt almost any position. The golden rule is to x-ray the pelvis in every case of severe
injury and, with femoral fractures, to insist on an x-ray that includes both the hip and knee.
The lower limb should be examined for signs of sciatic nerve injury.
Treatment
- Close reduction under general anaesthesia, but if this is not achieved after two or three
attempts an open reduction is required. An assistant steadies the pelvis; the surgeon starts by
applying traction in the line of the femur as it lies (usually in adduction and internal
rotation), and then gradually flexes the patient’s hip and knee to 90 degrees, maintaining
traction throughout. At 90 degrees of hip flexion, traction is steadily increased and
sometimes a little rotation (either internal or external) is required to accomplish reduction.
Another assistant can help by applying direct medial and anterior pressure to the femoral
head through the buttock. A satisfying ‘clunk’ terminates the manoeuvre. An important test
follows, to assess the stability of the reduced hip. By flexing the hip to 90 degrees and
applying a longitudinal and posteriorly-directed force, the hip is screened on an image-
intensifier looking for signs of subluxation. Evidence of this should prompt a repair to the
posterior wall of the acetabulum. Reduction is usually stable in type I injuries, but the hip
has been severely injured and needs to be rested. The simplest way is to apply traction and
maintain it for a few days. Movement and exercises are begun as soon as pain allows;
continuous passive movement machines are helpful. The terminal ranges of hip movements
are avoided to allow healing of the capsule and ligaments. As soon as active limb control is
achieved, and this may take about 2 weeks, the patient is allowed to walk with crutches but
without taking weight on the affected side. The rationale for not bearing weight is to prevent
collapse of femoral head due to an unsuspected avascular change. The period of hip
‘protection’ varies according to the risk of avascular necrosis: if the reduction was
performed promptly (within 6 hours), then no more than 6 weeks should suffice, but if there
was a longer delay then an extended period of 12 weeks may be wiser. Progression of
weightbearing should be graduated and the hip joint monitored by x-ray (Tornetta and
Mostafavi 1997). If the post-reduction x-rays or CT scans show the presence of intra-
articular bone fragments or larger femoral head pieces that are incompletely reduced, an
open procedure should be planned. The approach is dictated by the location of the fragment
on CT scan; however, the operation is not an emergency and can be done once the patient’s
condition has stabilized. The joint needs to be thoroughly washed out at the conclusion of
the procedure to remove bone ‘grit’. Type II fracture-dislocations are often treated by
immediate open reduction and anatomical fixation of the detached fragment, the rationale
being that many large posterior wall fragments either do not reduce well or remain as a
cause of instability even after reduction. However, if the patient’s general condition is
suspect, or the necessary surgical skills are not available, the hip is reduced closed, as
described above. Traction can be applied until conditions are appropriate for surgery – open
reduction and internal fixation will remedy the source of instability, return congruity to the
joint and remove any trapped bone fragments. Type III injuries are treated closed, but there
may be retained fragments and these should be removed by open operation. Fixation of a
comminuted posterior wall is sometimes impossible – if persistent instability is present,
referral to a specialist centre, where reconstruction using a segment of iliac crest could be
undertaken, is advisable. Types IV and V are treated initially by closed reduction. The
indications for surgery follow the principles already outlined: instability, retained fragments
or joint incongruity. In type V injuries, a femoral head fragment may automatically fall into
place, and this can be confirmed by post-reduction CT. If the fragment remains unreduced,
operative treatment is indicated: a small fragment can simply be removed, but a large
fragment should be replaced; the joint is opened, the femoral head dislocated and the
fragment fixed in position with a countersunk screw. Postoperatively, traction is maintained
for 2–4 weeks and full weightbearing is deferred for 12 weeks.

Complications
- Failed reduction multiple attempts at treatment, with failure to achieve concentric reduction,
may be worse than no treatment. The acetabulum remains undeveloped, the femoral head
may be deformed, the neck is usually anteverted and the capsule is thickened and adherent.
It is important to enquire also why reduction failed: is the dislocation part of a generalized
condition, or a neuromuscular disorder associated with muscle imbalance? The principles of
treatment for children over 8 years are the same as those discussed above. Avascular
necrosis A much-feared complication of treatment is ischaemia of the immature femoral
head. It may occur at any age and any stage of treatment and is probably due to vascular
injury or obstruction resulting from forceful reduction and hip splintage in abduction. The
effects vary considerably: in the mildest cases the changes are confined to the ossific
nucleus, which appears to be slightly distorted and irregular on x-ray. The cartilaginous
epiphysis retains the shape and physical growth is normal. After 12–24 months the
appearances return to normal. In more severe cases the epiphyseal and physeal growth plates
also suffer; the ossific nucleus looks fragmented, the epiphysis is distorted to greater or
lesser extent and metaphyseal changes lead to shortening and deformity of the femoral neck.
Prevention is the best cure: forced manipulative reduction should not be allowed; traction
should be gentle and in the neutral position; positions of extreme abduction must be
avoided; soft-tissue release (adductor tenotomy) should precede closed reduction; and if
difficulty is anticipated open reduction is preferable. Once the condition is established, there
is no effective treatment except to avoid manipulation and weightbearing until the epiphysis
has healed. In the mildest cases there will be no residual deformity, or at worst a femoral
neck deformity which can be corrected by osteotomy. In severe cases the outcome may be
flattening and mushrooming of the femoral head, shortening of the neck (with or without
coxa vara), acetabular dysplasia and incongruency of the hip. Surgical correction of the
proximal femur and pelvic osteotomy to reposition or deepen the acetabulum may be
needed.

37. Dislocation of shoulder joint and its management. Operative procedures for recurrent
shoulder dislocation.
Anterior shoulder dislocations usually result from abduction, extension, and external rotation, such
as when preparing for a volleyball spike. Falls on an outstretched hand are a common cause in older
adults. The humeral head is forced out of the glenohumeral joint, rupturing or detaching the anterior
capsule from its attachment to the head of the humerus or from its insertion to the edge of the
glenoid fossa. This occurs with or without lateral detachment.
Posterior dislocations are caused by severe internal rotation and adduction. This type of dislocation
usually occurs during a seizure, a fall on an outstretched arm, or electrocution. Occasionally, a
severe direct blow may cause a posterior dislocation. Bilateral posterior dislocation is rare and
almost always results from seizure activity. Misinterpretation of the radiograph appearance of a
posterior dislocation may result in misdiagnosis as a soft tissue injury in up to 79% of cases.
Rare, but serious, inferior dislocations (luxatio erecta) may be due to axial force applied to an arm
raised overhead, such as when a motorcycle collision victim tumbles to the ground. More
commonly, the shoulder is dislocated inferiorly by indirect forces hyperabducting the arm. The neck
of the humerus is levered against the acromion and the inferior capsule tears as the humeral head is
forced out inferiorly. Luxatio erecta almost always has an associated fracture or soft-tissue injury.
One series found 80% of patients to have fracture of the greater tuberosity or tear of the rotator cuff.
Neurologic compromise was found in 60% of patients, with the axillary nerve the most commonly
injured nerve. Inferior dislocations have the highest incidence (3.3%) of vascular compromise.
Management
In patients with shoulder dislocation, stabilize and treat associated trauma as indicated. Allow the
patient to assume a position of comfort while maintaining cervical spine immobilization if
necessary. A pillow between the patient's arm and torso may increase comfort. Administer
analgesics to decrease pain.Prereduction and postreduction radiographs are recommended. Patients
with frequent recurrent dislocations can safely avoid radiographs.Procedural sedation and analgesia
(PSA) protocols, intra-articular lidocaine, and ultrasound-guided brachial plexus nerve block assist
in making reduction an easier and more comfortable procedure. Using US-guided interscalene block
reduces time spent in the ED and lessens one-on-one health care provider time compared to
procedural sedation. Immobilize the shoulder after reduction.Perform careful prereduction and
postreduction neurovascular examinations. Orthopedic consultation may be helpful for dislocations
with concomitant fractures, for posterior or inferior dislocations, and for cases in which the patient's
shoulder cannot be reduced in a timely fashion.
In patients who have recurrent shoulder instability, operative care should be highly considered. The
goal of an operative repair is to reattach the torn tissue back to the place where it tore off of the
bone. Recurrent shoulder dislocations also stretch out the ligaments. It is imperative to also address
the tissue laxity during the operative procedure.

38. Classify fracture of neck of femur. Surgical anatomy of femur. Blood supply of the head of
femur. Treatment of fracture of neck of femur in adults. Clinical features, differential
diagnosis and management of fracture of neck of femur in postmenopausal women (***).
Garden stage I : undisplaced incomplete, including valgus impacted fractures medial group of
femoral neck trabeculae may demonstrate a greenstick fracture
Garden stage II : undisplaced complete no disturbance of the medial trabeculae
Garden stage III : complete fracture, incompletely displaced femoral head tilts into a varus
position causing its medial trabeculae to be out of line with the pelvic trabeculae
Garden stage IV : complete fracture, completely displaced femoral head aligned normally in the
acetabulum and its medial trabeculae are in line with the pelvic trabeculae
In general, stage I and II are stable fractures and can be treated with internal fixation (head-
preservation) and stage III and VI are unstable fractures and hence treated with arthroplasty (either
hemi- or total arthroplasty)
39. Stages, clinical features, investigations and management of tuberculosis of hip joint.
Stages CF Investigation and Management.
Clinical Feature:
• Insidious onset • Chronic course • Commonly associated with constitutional symptoms • Limp or
lameness – earliest and commonest sign , antalgic gait • Pain - absent in early stage • Night cries •
Decreased ROM • Fullness around the hip – cold abscess Deformity - depending on the stage
Wasting of the thigh and gluteal muscle Limb length discrepency Due to fixed deformity Secondary
changes – lordosis , scoliosis
Stages
1. Stage of synovitis : • Irritable hip , painful movements • Flexion , abduction , external rotation .
(FABER ) apparent lengthening. • X-RAY – soft tissue swelling , haziness of articular margins &
rarefaction • USG – soft tissue swelling • MRI – synovial effusion • Biopsy – can be done for
confirmation
2. Stage of early arthritis : Destruction of articular cartilage Spasm of adductors , flexors + wasting
Flexion , adduction , internal rotation (FADIR) , Apparent shortening • ↓sed ROM • X-RAY –
osteopenia , erosion of articular margins , ↓ joint space • MRI - synovial effusion , oedema ,
minimal bone destruction
3. Stage of advanced arthritis : Further destruction of joint True shortening > 1 cm Muscle wasting
Decrease in ROM X RAY - further decrease in joint space.
4. Advanced arthritis with subluxation / dislocation : • Further destruction of acetabulum , head ,
capsule and ligaments. • Gross restriction of ROM • Head – upwards and posteriorly • Wandering /
migrating acetabulum • Reduced joint space

Investigation
o CBC
o ESR
o MANTOUX TEST
o TB ELISA
o IMAGING : X RAY – HIP
o SYNOVIAL BIOPSY
o SMEAR , CULTURE AND GUINEA PIG INOCULATION
o MRI - EFFUSION , SYNOVIAL THICKENING , JUXTRA ARTICULAR OSTEOPENIA
o PCR – RT PCR , NESTED PCR

Management.
o General Treatment: Good Diet, Fresh Air, Sunlight, Education, Occupation
o CHEMOTHERAPY • 6-9 MONTH DURATION • 12 MONTH FOR PEDIATRIC AGE
GROUP
o Drugs: 1st line - HRZES • 2ND Line Capreomycin Kanamycin Ethionamide Cycloserine
PAS • Newer drugs : rifapentin , rifabutin , gatifloxacin , moxifloxacin
o Bactericidal drugs 1.Isoniazid 2. Rifampicin Dose 5mg/kg 10-15 mg/kg 3. Streptomycin
20mg/kg 4. Pyrazinamide 20-25 mg/kg Bacteriostatic drugs 1. Ethambutol Dose 25mg/kg (x
2mnths) Then 15mg/kg

40. Clinical features, investigations and management of Ewing’s sarcoma (*)


CF
• 5 - 25yrs. (90% below 20)
• Highest frequency 5-15yrs
• Mean age - 13yrs
• Male predominance – 3:2
• Localised, painful, tender mass
• Systemic symp. – fever, malaise, weight loss - mistaken for OM - dissemination of tumor
• May metastasize to other bones 9

Investigation
-MRI
-CT Scan
-Plain Radiography
-Radiostope bone scan
Management.
-Chemotheraphy
-Surgery
-Radiotherapy

41. Fractures and dislocations caused by fall on an outstretched hand. Supracondylar fracture
of humerus in child. (**)

Classification
Gartland Classificaiton
(may be extension or flexion type)
Type I
Nondisplaced
beware of subtle medial comminution leading to cubitus varus, which technically means it is not
a Type 1 Fracture, and it requires reduction and pinning.
Treated with cast immobilization x 3-4wks, with radiographs at 1 wk
Type II
Displaced
posterior cortex and posterior periosteal hinge intact
Deformity is in the sagittal plane only
Typically treated with CRPP

Type III
Displaced, often in 2 or 3 planes
Treated most commonly with CRPP or open reduction if needed

Type IV**
Complete periosteal disruption with instability in flexion and extension
Diagnosed with examination under anesthesia during surgery
Treated most commonly with CRPP or open reduction if needed
SCH flexed

Presentation
Symptoms
Pain, refusal to move the elbow
Physical exam
Inspection-gross deformity, swelling, ecchymosis in ante-cubittal fossa, limited active elbow
motion
neurovascular exam must be done before any reduction maneuver to be certain nerve or vascular
injury is not iatrogenic (stuck in fracture site).
Imaging
Radiographs
recommended views-AP and lateral x-ray of the elbow (really of the distal humerus)
findings- posterior fat pad sign, lucency on a lateral view along the posterior distal humerus and
olecranon fossa is highly suggestive of occult fracture around the elbow measurement
Treatment
Nonoperative- long arm casting with less than 90° of elbow flexion, typically used for 3 weeks
repeat radiographs at 1 week to assess for interval displacement
Operative -losed reduction and percutanous pinning (CRPP)
42. Pathophysiology, clinical features and investigations and management of trochanteric
fractures of femur.
Fractures in the inter-trochanteric region of the proximal femur, involving either the greater or the
lesser trochanter or both, are grouped in this category. In the elderly, the fracture is normally
sustained by a sideway fall or a blow over the greater trochanter. In the young, it occurs following
violent trauma, as in a road traffic accident. The distal fragment rides up so that the femoral neck-
shaft angle is reduced (coxa vara). The fracture is generally comminuted and displaced. Very rarely,
it can be an undisplaced fracture.
CF: Pain in the region of the groin and an inability to move the leg. There will be swelling in the
region of the hip, and the leg will be short and externally rotated. There is tenderness over the
greater trochanter
IX: X-ray: Presence of comminution of the medial cortex of the neck, avulsion of the lesser
trochanter and extension of the fracture to the subtro-chanteric region indicate an unstable fracture.
MANAGEMENT: The main objective of treatment is to maintain a normal femoral neck-shaft
angle during the process of union. This can be done by conservative means (traction) or by internal
fixation.

43. Define and classify osteomyelitis. Pathogenesis, clinical features, complications and
management of chronic nonspecific osteomyelitis.
Definition: Infection of bone characterized by progressive inflammatory destruction and apposition
of new bone
CLASSIFICATION:
Anatomic Location
 Stage I- Medullary,
 Stage 2-Superficial,
 Stage 3-Localized,
 Stage 4-Diffuse
Host Type
 Type A Normal,
 Type B Compromised,
 Type C Treatment is worse to patient than infection
PATHOLOGY
Acute osteomyelitis commonly leads to chronic osteomyelitis because of one or more of the
following reasons:
a) Delayed and inadequate treatment: This is the commonest cause for the persistence
of an osteomyelitis. Delay causes spread of pus within the medullary cavity and subperiosteally.
This results in the death of a part of the bone (sequestrum formation). Destruction of cancellous
bone leads to the formation of cavities within the bone. Such ‘non-collapsing’ bone cavities and
sequestra are responsible for persistent infection.
b) Type and virulence of organism: Sometimes, despite early, adequate treatment of acute
osteomyelitis, the body’s defense mechanism may not be able to control the damaging
influence of a highly virulent organism, and the infection persists.
c) Reduced host resistance: Malnutrition compromises the body’s defense mechanisms,
thus letting the infection persist.
CLINICAL FEATURES: Chronic discharging sinus, thickened, irregular bone, stiffness of joint,
tenderness on palpation.
COMPLICATIONS: Persistence or extension of infection, Amputation, Sepsis, Malignant
transformation (Marjolin's ulcer)

44. Clinical features, investigations and management of osteoclastoma. (***)


Osteoclastoma is a tumor of bone characterized by massive destruction of the end (epiphysis) of a
long bone. The most common bones involved are the long bones; less common are the jaws,
vertebrae, scapula pelvis and small bones of hands and feet.

A)Clinical Features

I) present with pain


ii) limited range of motion
iii). Swelling may occur, as well, if the tumor has been growing for a long time.
iv)muscular aches and pains in arms or legs and abdominal pain
v)may also experience nerve pain which feels like an electric shock due to weight bearing.

B) Investigation
i)biopsy findings- histomorphologic feature is (multinucleated) giant cells with up to a hundred
nuclei that have prominent nucleoli. Surrounding mononuclear and small multinucleated cells have
nuclei similar to those in the giant cells; this distinguishes the lesion from other osteogenic lesions
which commonly have (benign) osteoclast-type giant cells.

ii) X-ray- Giant-cell tumors (GCTs) are lytic/lucent lesions that have an epiphyseal location and
grow to the articular surface of the involved bone.Radiologically the tumors may show
characteristic 'soap bubble' appearance.They are distinguishable from other bony tumors in that
GCTs usually have a nonsclerotic and sharply defined border.

C) Management
i) Surgery- Curettage, in medical procedures, to remove tissue by scraping or scooping.
ii)Pharmacotherapy for GCTOB, includes bisphosphonates such as Zoledronate, which are thought
to induce apoptosis in the MNGC fraction, preventing tumor-induced osteolysis
iii) immobilised the affected limb(pathological fracture)

45. List injuries around the elbow joint. (*)


a) Supracondylar Fracture of Humerus
b)Cubitus Varus
c)Lateral Humeral Condyle Fracture
d) Cubitus Valgus
e)Radial head Fracture
f)Elbow dislocation
g)Pulled elbow
h) Medial epicondyle fracture

46. Mechanism, classification and management of ankle fractures


 Lauge-Hansen

o based on foot position and force of applied stress/force


o has been shown to predict the observed (via MRI) ligamentous injury in less than
50% of operatively treated fractures
Lauge- Sequence
Hansen
Class
Supination - 1. Talofibular sprain or distal fibular avulsion
Adduction 2. Vertical medial malleolus and impaction of anteromedial distal tibia

(SA)

Supination - 1. Anterior tibiofibular ligament sprain


External 2. Lateral short oblique fibula fracture (anteroinferior to posterosuperior)
Rotation 3. Posterior tibiofibular ligament rupture or avulsion of posterior malleolus
(SER) 4. Medial malleolus transverse fracture or disruption of deltoid ligament

Pronation - 1. Medial malleolus transverse fracture or disruption of deltoid ligament


Abduction
2. Anterior tibiofibular ligament sprain
(PA)
3. Transverse comminuted fracture of the fibula above the level of the

syndesmosis

Pronation - 1. Medial malleolus transverse fracture or disruption of deltoid ligament


External
2. Anterior tibiofibular ligament disruption
Rotation
3. Lateral short oblique or spiral fracture of fibula (anterosuperior to
(PER)
posteroinferior) above the level of the joint

4. Posterior tibiofibular ligament rupture or avulsion of posterior malleolus

Classification

The Weber ankle fracture classification (or Danis-Weber classification) is a simple system for
classification of lateral malleolar fractures, relating to the level of the fracture in relation to
the ankle joint. It has a role in determining treatment

 type A
o below the level of the talar dome
o usually transverse
o tibiofibular syndesmosis intact
o deltoid ligament intact
o medial malleolus occasionally fractured
o usually stable if medial malleolus intact
 type B
o distal extent at the level of the talar dome; may extend some distance proximally
o usually spiral
o tibiofibular syndesmosis usually intact, but widening of the distal tibiofibular joint (especially
on stressed views) indicates syndesmotic injury
o medial malleolus may be fractured
o deltoid ligament may be torn, indicated by widening of the space between the medial
malleolus and talar dome
o variable stability, dependent on the status of medial structures (malleolus/deltoid ligament)
and syndesmosis; may require ORIF
o Weber B fractures could be further subclassified as
 B1: isolated
 B2: associated with a medial lesion (malleolus or ligament)
 B3: associated with a medial lesion and fracture of posterolateral tibia
 type C
o above the level of the ankle joint
o tibiofibular syndesmosis disruption with widening of the distal tibiofibular articulation
o medial malleolus fracture or deltoid ligament injury often present
o fracture may arise as proximally as the level of fibular neck and not visualised on ankle films,
requiring knee or full-length tibia-fibula radiographs (Maisonneuve fracture)
o unstable: usually requires ORIF
o Weber C fractures can be further subclassified as 6
 C1: diaphyseal fracture of the fibula, simple
 C2: diaphyseal fracture of the fibula, complex
 C3: proximal fracture of the fibula
 fracture above the syndesmotic result from external rotation or abduction forces that
also disrupt the syndesmosis
 usually associated with an injury to the medial side
Management
o Stabilize the suspected fracture site with a pillow splint, air splint, or bulky Jones dressing
before transporting patient.
o Nonoperative
o short-leg walking cast/boot
o Operative
o open reduction internal fixation

47. Classify traumatic dislocation of hip. Describe mode of injury and management of each.
Classify- Anterior, posterior and central dislocations.

Mode- Anterior: The femoral head being displaced to come to lie anterior to the coronal plane of
the acetabulum. This results in the leg assuming an extended and externally rotated position.

Posterior: Direct longitudinal force applied to the femur and hip. Consequently, the femoral
head comes to lie posterior to the acetabulum in the coronal plane with the hip assuming a
semiflexed internally rotated position.

Central: Traumatic injury where femoral head is pushed through acetabulum toward pelvic
cavity.
Management- CT scan or MRI scan
- Closed reduction if known no fractures

48. Clinical features, manifestations and complications of rheumatoid arthritis.


Clinical features-
Stiffness, Tenderness, Pain on motion, Swelling, Deformity, Limitation of motion, Extra-articular
manifestations, Rheumatoid nodules.

Manifestations-
Tender, warm, swollen joints.
Joint stiffness that is usually worse in the mornings and after inactivity.
Fatigue, fever and weight loss

Complications- Rheumatoid nodules, vasculitis, conjunctivitis, pleuritis, palmar erythema,


amyloidosis

49. CTEV: Congenital Talipes Equinovarus also known as club foot


Idiopathic deformity of the foot of unclear etiology

Pathophysiology
Muscle contractures contribute to the characteristic deformity that includes (CAVE)
 Cavus (tight intrinsics, FHL, FDL)
 Adductus of forefoot (tight tibialis posterior)
 Varus (tight tendoachilles, tibialis posterior, tibialis anterior)
 Equinus (tight tendoachilles)
Bony deformity consists of medial spin of the midfoot and forefoot relative to the hindfoot, talar
neck is medially and plantarly deviated, calcaneus is in varus and rotated medially around talus,
navicular and cuboid are displaced medially, table of foot deformity muscle imbalances
Presentation
Physical exam
Inspection- small foot and calf , shortened tibia, medial and posterior foot skin creases, foot
deformities, hindfoot in equinus and varus, differentiated from more common positional foot
deformities by rigid equinus, and resistance to passive correction, midfoot in cavus, forefoot in
adduction
Imaging
Radiographs
-dorsiflexion lateral (Turco view)
-AP
Ultrasound - Clubfoot sometimes diagnosed in utero
Treatment
Nonoperative- Ponseti method of serial manipulation and casting
Operative- posteromedial soft tissue release and tendon lengthening

Potrebbero piacerti anche